Sunteți pe pagina 1din 72

Din sumar :

* Articole ]i note matematice


* Concursuri ]i olimpiade
* 100 de probleme propuse  pentru clasele I - IV
* 175 de probleme teme  pentru clasele V - XI
* Lucr[ri de verificare  pentru clasele V - XII
* Probleme propuse  pentru clasele V - XII
* Rubrica rezolvitorilor

Anul XXVII , Nr. 1


2016 / 2017
R.M.C.

Sumar
1. ARTICOLE }I NOTE MATEMATICE
 PARTI|II #N INEGALIT{|I - prof. Leonard Giugiuc, Daniel Sitaru, Diana Tr[ilescu . . . . . . . . 1
 SOME GENERALIZATIONS OF SOME PROBLEMS FROM SCHOOL SCIENCE AND MATHE-
MATICS - prof. D. M. B[tine\u-Giurgiu, Neculai Stanciu . . . . . . . . . . . . . . . . . . . . . . . . . . . . . . . . . 4
 OBSERVA|II ASUPRA UNOR NO|IUNI DIN GEOMETRIA TRIUNGHIULUI - prof. Otilia
Dr[gan . . . . . . . . . . . . . . . . . . . . . . . . . . . . . . . . . . . . . . . . . . . . . . . . . . . . . . . . . . . . . . . . . . . . . . . . . . 6
2. CONCURSURI }I OLIMPIADE
 CONCURSUL DOLJEAN DE MATEMATIC{ - 19 martie 2016 . . . . . . . . . . . . . . . . . . . . . . . . . . . . 9

3. PROBLEME PROPUSE  CICLUL PRIMAR . . . . . . . . . . . . . . . . . . . . . . . . . . . . . . . . . . 19


4. PROBLEME - TEME
 Gimnaziu . . . . . . . . . . . . . . . . . . . . . . . . . . . . . . . . . . . . . . . . . . . . . . . . . . . . . . . . . . . . . . . . . . . . . . . 28

 Liceu . . . . . . . . . . . . . . . . . . . . . . . . . . . . . . . . . . . . . . . . . . . . . . . . . . . . . . . . . . . . . . . . . . . . . . . . . . 36

5. LUCR{RI DE VERIFICARE  Clasele V - XII . . . . . . . . . . . . . . . . . . . . . . . . . . . . . . . . . . 47


6. PROBLEME PROPUSE
 Gimnaziu . . . . . . . . . . . . . . . . . . . . . . . . . . . . . . . . . . . . . . . . . . . . . . . . . . . . . . . . . . . . . . . . . . . . . . . 62

 Liceu . . . . . . . . . . . . . . . . . . . . . . . . . . . . . . . . . . . . . . . . . . . . . . . . . . . . . . . . . . . . . . . . . . . . . . . . . . 65

7. RUBRICA REZOLVITORILOR . . . . . . . . . . . . . . . . . . . . . . . . . . . . . . . . . . . . . . . . . . . . . 68

Redac\ia R.M.C.
Director editor : prof. D[nu\ Dr[cea
Redactor coordonator : prof. Dan Secl[man
Colectivul de redac\ie : @nv. Mariana Ungureanu, prof. Constantin Basarab, prof. Constantin
Cazacu, prof. Dorina Dr[cea,prof. Liliana Niculescu, prof. Ion P[tra]cu,
prof. Daniel Al. Ion, prof. Nicolae T[l[u, prof. Lucian Tu\escu,
prof. dr. Grigorie Preoteasa, prof. Otilia Dr[gan, prof. Felician Preda,
prof. Octavian Ungureanu, prof. C[t[lin Cristea, prof. Raluca Ciurcea,
prof. Cristian Paul Moan\[, prof. Cerasela Bociu, prof. Delia Mitran.
Colaboratori : prof. Dan Popescu (Suceava), prof. Traian Ianculescu (Zimnicea), prof. Emilia Velcea
(Lupeni), prof. univ. dr. Dumitru Acu (Sibiu), prof. univ. dr. Cristinel Mortici
(T`rgovi]te), prof. univ. dr. Florentin Smarandache (New Mexico, Arizona, U.S.A),
prof. dr. Dorin M[rghidanu (Corabia).
Adresa redac\iei: EDITURA CARDINAL:
Str. R[zboieni, Nr. 2, Bl. B13, Sc. 1, Ap. 2, Craiova, Cod po]tal: 200667
Telefon: 0744.555376; 0351.422424 Fax: 0351.422425

© Editura CARDINAL
 Tehnoredactare computerizat[:
E

All Rights Reserved C Irina Liliana Suciu (Editura Cardinal)


REVISTA DE MATEMATIC{ DIN CRAIOVA 

Publica\ie semestrial[ pentru elevi ]i profesori

 E ANUL XXVII Nr. 1


anul ]colar 2016/2017
C

* ARTICOLE }I NOTE MATEMATICE *


***************************************************************************

 1. PARTI|II #N INEGALIT{|I
Prof. LEONARD GIUGIUC, Dr. Tr. Severin
Prof. DANIEL SITARU, Dr. Tr. Severin
Prof. DIANA TR{ILESCU, Dr. Tr. Severin

#n cele ce urmeaz[ autorii vor prezenta o metod[ original[ de demonstrare a unor


inegalit[\i.
Consider[m n variabile nenegative av`nd suma egal[ cu nk, unde n este @ntreg,
n  3 ]i k este un num[r pozitiv fixat. Vom parti\iona intervalul de lucru 0, nk @n
func\ie de pozi\ia variabilelor din list[ @n raport cu media lor aritmetic[, k.
Pentru a putea explica mai bine metoda, vom demonstra @n cele ce urmeaz[ un rezultat
cu un grad ridicat de complexitate.

Teorem[. (Vasile Cirtoaje)


Dac[ a, b, c, d sunt numere reale nenegative av`nd suma pozitiv[, atunci:
(a  b  c  d )(a  b  c  d )(a  b  c  d )(a  b  c  d ) 
 (a  b  c  d )(abc  abd  acd  bcd ).
Demonstra\ie. Cum ambii membri ai inegalit[\ii sunt func\ii simetrice ]i omogene
de gradul 4, putem presupune c[ a  b  c  d  4 ]i a  b  c  d. De aici ob\inem c[
a  1  d ]i inegalitatea echivalent[:
16(ab  bc  cd  da  ac  bd )  4abcd  64  9(abc  abd  acd  bcd ).
Vom crea urm[toarele parti\ii:
1) a  1  b  c  d  0; 2) a  b  c  1  d  0; 3) a  b  1  c  d  0.
Cazul 1: a  1  b  c  d  0.
Not[m ab  bc  cd  da  ac  bd  q, bcd  abd  acd  abc  s, abcd  p ]i conside-
r[m func\ia polinomial[ f : R R, f (x )  (x  a )(x  b )(x  c )(x  d ).
Atunci f (x )  x 4  4x 3  qx 2  sx  p ]i avem f  (x )  4x 3  12x 2  2qx  s. Din teorema
lui Rolle exist[ numerele reale   d, c,   c, b ]i   b, a astfel @nc`t ,  ]i  sunt
r[d[cinile lui f  . De aici deducem c[ 1      0 ]i cum       3 ob\inem 1  .
Deoarece f  (x )  4(x   )(x   )(x   ) deducem c[ f  (1 )  0 de unde 2q  s  8, .
Inegalitatea de domonstrat este 16q  4p  64  9s ]i cum din (  ) avem 16q  8s  64,
este suficient s[ ar[t[m c[ 4p  s, adic[: 4abcd  abc  abd  acd  bcd.
3
Conform inegalit[\ii mediilor avem 4(abcd ) 4  abc  abd  acd  bcd ]i abcd  1, iar
3
de aici ob\inem abcd  (abcd ) 4 , de unde 4abcd  abc  abd  acd  bcd ]i demon-
stra\ia acestui caz este @ncheiat[.
S[ remarc[m c[ egalitatea are loc dac[ ]i numai dac[: a  b  c  d  1.
Cazul 2: a  b  c  1  d  0.
Not[m a  1  x, b  1  y, c  1  z, d  1  t, avem x, y, z  0, 0  t  1, iar x  y  z  t.
Dac[ t  0 atunci x  y  z  0, deci a  b  c  d  1, ceea ce s-a studiat deja. Pentru
t  0, 1 avem ab  bc  cd  da  ac  bd  6  t 2  xy  yz  zx, abcd  (1  t )1  t 
xy  yz  zx  xyz ]i abc  abd  acd  bcd  1  t  xy  yz  zx  xyz  (1  t )3  2t 
xy  yz  zx. Deci, inegalitatea de demonstrat devine: 2t 2  (2  5t )(xy  yz  zx ) 
(5  4t )xyz  0. #n continuare vom face apel la articolul Aplica\ii ale func\iei cubice,
publicat @n R.M.T. Nr. 1/2014. Not[m t  3s ]i atunci 0  s  1 . Deci exist[ u  0, s
3
cu proprietatea c[ xy  yz zx  3(s 2  u 2 ) ]i fie xyz  p.
Subcazul 1: 0  u  s .
2
Valoarea minim[ a lui p se atinge @n x  y  s  u ]i z  s  2u ]i este egal[ cu: 2u 3 
3su 2  s 3 . Deci e suficient de ar[tat c[ 18s 2  3(2  15s )(s 2  u 2 )  (5  12s )2u 3 
3su 2  s 3   0, adic[: 2(5  12s )u 3  6(6s 2  5s  1 )u 2  4s 2 (3s 2  10s  3 )  0.
Pentru fiecare s fixat @n intervalul 0, 1 fie func\ia f : 0, s  R, f (u )  25 
3 2
12su 3  6(6s 2  5s  1 )u 2  4s 2 (3s 2  10s  3 ).
(6s 2  5s  1 ) s
Avem f  (u )  6u(5  12s )u  2(6s 2  5s  1 ) ]i cum  , ( )s 
5  12s 2
 0, 1 , deducem c[ f este strict cresc[toare, de unde: min f  f (0 )  4s 2 (1  3s )3 
3
s  0, ( ) s  0, 1 .
3

2 Revista de matematic[ din Craiova


Subcazul 2: s  u  s.
2
Atunci: 18s 2  3(2  15s )(s 2  u 2 )  (5  12s )p  18s 2  3(2  15s )(s 2  u 2 ).
Deoarece func\ia g(u )  18s 2  3(2  15s )(s 2  u 2 ) este strict cresc[toare pentru
s  u  s, ob\inem: min g  g s  18s 2  9 s 2 (2  15s )  0, ( )s  0, 1 ]i deci de-
2 2 4 3
monstra\ia cazului 2 a fost @ncheiat[.
Egalitatea are loc pentru a  b  c  4 ]i d  0.
3
Cazul 3: a  b  1  c  d  0.
Not[m a  1  x, b  1  y, c  1  z, d  1  t ]i avem x, y, z, t  0, x  y  z  t  2s ]i
z, t  1 de unde s  0, 1. Fie xy  p ]i zt  q. Cum cazul s  0 este trivial, el fiind deja
studiat, putem considera 0  s  1; avem: ab  bc  cd  da  ac  bd  6  4s 2  p  q;
abcd  1  4s 2  p  q  2s(q  p )  pq; abc  abd  acd  bcd  4  8s 2  2p  2q 
2s(q  p ). Deci, inegalitatea de demonstrat devine: 4s 2  (p  q )  5s(q  p )  2pq 
 0  p(1  5s  2q )  4s 2  q(5s  1 )  0. Dar 1  5s  2q  0 ]i p  s 2 , de unde
p(1  5s  2q )  4s 2  q(5s  1 )  s 2 (1  5s  2q )  4s 2  q(5s  1 ). Deci, este sufici-
ent s[ ar[t[m c[ s 2 (1  5s  2q )  4s 2  q(5s  1 )  0, ceea ce este echivalent cu:
q(2s 2  5s  1 )  3s 2  5s 3  0.
5  17
Subcazul 1: 0  s  . Atunci 2s 2  5s  1  0 ]i q  s 2, de unde rezult[:
4
q(2s 2  5s  1 )  3s 2  5s 3  s 2 (2s 2  5s  1 )  3s 2  5s 3  0.
5  17
Subcazul 2:  s  1.
4 2
Atunci 2s 2  5s  1  0 ]i q  0, de unde: q(2s 2  5s  1 )  3s 2  5s 3  3s 2  5s 3  0.
Subcazul 3: 1  s  1.
2
Cum z  1 ]i 2s  z  1, deducem c[ z, t  2s  1, 1, de unde rezult[: min q  2s  1.
Deoarece 2s 2  5s  1  0 ]i q  2s  1, ob\inem:
q(2s 2  5s  1)  3s 2  5s 3  (2s  1 )(2s 2  5s  1 )  3s 2  5s 3  (1  s )(3s  1 ) 2  0.
Deci demonstra\ia cazului 3 ]i implicit a teoremei, au fost finalizate.
S[ remarc[m c[ al treilea caz de egalitate are loc @n (2, 2, 0, 0 ).
Ca o concluzie de final, @n cazul general egalitatea se atinge @n permut[rile cua-
druplelor (, , ,  ), (, , , 0 ) ]i (, , 0, 0 ), unde   0.
#n @ncheiere, vom propune cititorului spre rezolvare trei aplica\ii ale c[ror rezolv[ri
sunt pretabile la metoda utilizat[ @n demonstra\ia de mai sus.
Aplica\ia 1. (Vasile Cartoaje):
Fie numerele reale nenegative a, b, c ]i d, av`nd suma pozitiv[.
Demonstra\i inegalitatea:
27(a  b  c  d )(abc  abd  acd  bcd )  (a  b  c  d ) 4  176abcd.
Aplica\ia 2. (Al patrulea test de selec\ie pentru BMJO 2016, autor Leonard Giugiuc):
Fie numerele reale a, b, c astfel @nc`t a  b  1  c  0 ]i a  b  c  3.
(a ) S[ se arate c[: 2  ab  bc  ca  3.

Editura Cardinal 3
(b ) Demonstra\i inegalitatea 3 243  25  14 ]i preciza\i cazurile de
a  b  c 3 ab  bc  ca
egalitate.
Aplica\ia 3. (Crux Mathematicorum, autori Leonard Giugiuc ]i Daniel Sitaru):
Fie a, b, c ]i d numere reale cu a  b  c  1  d  0 ]i a  b  c  d  4.
Demonstra\i c[: 3 98  363  80 ]i preciza\i c`nd
a  b 3  c 3  d 3 ab  bc  cd  da  ac  bd
are loc egalitatea.

Bibliografie:
1 Leonard Giugiuc, Aplica\ii ale func\iei cubice, RMT, Nr. 1/2014.
2 Daniel Sitaru, Leonard Giugiuc, Claudia Nanuti, Diana Tr[ilescu,
Inequalities, Editura Ecko Print, 2015.
3 Radu Gologan, Daniel Sitaru, Leonard Giugiuc, 300 Romanian Challenges, Editura
Paralela 45, 2016.
4 Daniel Sitaru, Math Phenomenon, Editura Paralela 45, 2016.
5 Crux Mathematicorum.

*
 2. SOME GENERALIZATIONS OF SOME PROBLEMS FROM
SCHOOL SCIENCE AND MATHEMATICS

Prof. D. M. B{TINE|U-GIURGIU, Bucure]ti


Prof. NECULAI STANCIU, Buz[u

In the following we give some generalizations of some problems from School


Science and Mathematics journal, April 2016.

Proposition. If t  R and (an ) n1 , (bn ) n1 be positive real sequences such that
a n1 b n1
n1
a n1 b n1 n
anbn
lim na  a  R 
 , lim  b  R 
 , then: lim   ab .
n n n n b n
t n (n  1 ) t nt e t1
anbn n
anbn a n1 b n1 n n(t1) 
Proof. We have lim
n t1 lim lim (n1 )(t1 )  a b
n
n n n n(t1) n (n  1 ) n n
(n1 )(t1 )
a n1 b n1 n ab .
lim    a  b  e1t1 
n na n ntbn n  1 e t1
a n1 b n1 a n1 b n1 n t1  n  1 ,
n1 n1

If u n   n t , ( )n  2, then u  
(n  1 ) t n
(n  1 ) t1 n
n
anbn n
anbn
lim u  ab  e t1  1  1, so lim u n  1  1.
n
n e t1 ab n ln u n

4 Revista de matematic[ din Craiova


nt
a n1 b n1 nt a n1 b n1
We have lim u n lim  n nt  1 lim  t  n 
n n n anbn (n  1 ) n1
a n1 b n1 n na n n bn n1

n t1 n ntt1 (n  1 ) t1 (n  1 ) t1


  a  b lim
n n  1
  a  b  e t
lim
n n1

n1
a n1 b n1 n1
a n1 b n1 a n1 b n1
t1 n1
a n1 b n1 n
anbn n
anbn
 abe t  e  e. We denote B n   nt  n t (u n  1 ) 
ab (n  1 ) t

n
anbn u  1 n
anbn un  1
 nt  n  ln u n    ln u nn , ( )n  2.
ln u n n t1 ln u n
anbn un  1
n

Hence, lim B
n n n
 lim n t1 lim
n ln u n
 ln lim u n  eab
n n
ab
t1  1  ln e  e t1 .

Remark. If we take above t  1, a n  n!, b n  (2n  1 )!! , then by Proposition 1 we


a n1 (n  1 )! n  1  1, and lim (2n  1 )! lim 2n  1  2,
obtain lim lim lim
n na n n n  n! n n n n  (2n  1 )!! n n

(n  1 )!(2n  1 )!!
n1 n
n!(2n  1 )!! 2  2 ,
i.e. a  1, b  2.So, lim   ab  1 11
n n1 n e t1 e e2
i.e. the problem 5398 from School Science and Mathematics.
Proposition 2. If a, b, c, m, n, p  R , then:
m(a  b ) 3m .
 ( 
np
cyclic n  2p )a  2nab  (n  2p )b 2
2

Proof. We have (n  2p )a 2  2nab  (n  2p )b 2  n(a 2  2ab  b 2 )  2p(a 2  b 2 ) 


(a  b ) 2
 n(a  b )  2p(a 2  b 2 )  n(a  b )  2p   (n  p )(a  b ) .
2 2 2
2
m(a  b ) m(a  b )
There fore,   
cyclic (n  2p )a 2  2nab  (n  2p )b 2 cyclic (n  p )(a  b ) 2
(a  b )
m   3m .
cyclic np  ab( ) np
Remark. If we take m  n  p  2, then by Proposition 2 we obtain the inequality
from Problem 5399 from School Science and Mathematics Journal, April, 2016,
proposed by Angel Plaza.
b 1 3
Proposition 3. If a, b, c, m, n  R  , then:  2 
(m  n ) 2
.
cyclic
m a n b
Proof. We have
b 1 1 1 AMGM
 2   2   2 
cyclic
m a n b cyclic
b m a n b cyclic
m ab  nb
AMGM 1 1 1 Radon
  m(ab ) 2 4 
cyclic (m(a  b )  2nb )
2 4 
cyclic (ma  (m  2n )b )
2 
cyclic
2  nb

Editura Cardinal 5
Radon (1  1  1 ) 3 27
 4 2 4 2 

cyclic
(ma  (m  2n )b ) m 
cyclic
a  (m  2n ) 
cyclic
b

4 27 27
2  , (  ).
4 (m  n ) (a  b  c ) (m  n ) 2 (a  b  c ) 2
2

(a  b  c ) 2
By hypothesis we deduce that: 3  a 2  b 2  c 2   9  (a  b  c ) , (  ).
2
3
By (  ) and (  ) we obtain:  b 1 27  3
2  .
cyclic
m a n b (m  n )  9 (m  n ) 2
2

Remark. If we take m  4, n  3, then by Proposition 3 we obtain the problem 5401


from School Science and Mathematics.

*
 3. OBSERVA|II ASUPRA UNOR NO|IUNI
DIN GEOMETRIA TRIUNGHIULUI
Prof. OTILIA DR{GAN, Craiova

Fie un triunghi ABC, @n care not[m cu a, b, c lungimile laturilor (BC ), (AC ) ]i


respectiv (AB ) ]i cu h a , h b , h c lungimile @n[l\imilor din A, din B, respectiv, din C. Vom
stabili @n ce condi\ii, numerele h a , h b , h c pot fi lungimile laturilor unui triunghi. Dac[
a  ha b  hb c  hc
not[m cu S, aria triunghiului ABC, vom putea scrie S    , de
2 2 2
unde deducem: h a  a , h b  2S 2S 2S
, h c  c . Pentru a exista un triunghi cu laturile
b
h a , h b , h c trebuie ca
 2S  2S  2S  1  1  1
 h a  h b  h c  a b c  a b c  bc  ac  ab
  2S 2S 2S  1 1 1 
 h b  h a  h c   b  c  a   b  c  a   ac  ab  bc , (1 )
 h  h  h  2S 2S 2S  1 1 1  ab  bc  ac
 c a b   a    a 
 c b  c b
A]adar, pentru a exista triunghiul precizat, trebuie s[ aib[ loc inegalit[\ile (1):
Exemple. 1) Fie triunghiul ABC, cu AB  2, AC  b  3, Ab  c  4. Acest triunghi
 a  b  c  2  3  4
 
exist[, c[ci:  b  c  a ,   3  4  2 . Verific[m dac[ au loc inegalit[\ile (1);
 c  a  b  4  2  3
 
 3  4  2  4  2  3  12  8  6
 
avem  2  4  2  3  3  4 , adic[  8  6  12 ]i deducem c[ exist[ un triunghi ale
 2  3  3  4  2  4  6  12  8
 
c[rui laturi sunt congruente cu @n[l\imile triunghiului ABC.
2) Consider[m triunghiul ABC cu BC  a  2, AC  b  9, Ab  c  10.

6 Revista de matematic[ din Craiova


Se verific[ u]or c[ exist[ un astfel de triunghi ]i deoarece, inegalit[\ile (1) nu sunt
verificate, concluzion[m c[ nu exist[ un triunghi ale c[rui laturi s[ fie congruente cu
@n[l\imile triunghiului ABC.
#n continuare, vom ar[ta, c[ dac[ exist[ un triunghi ale c[rui laturi sunt congruente
cu @n[l\imile unui triunghi dat ]i acest triunghi este asemenea cu triunghiul dat, atunci
@n fiecare din cele dou[ triunghiuri, una dintre laturi este media geometric[ a celorlalte
laturi. Consider[m triunghiul ABC, @n care a, b, c sunt lungimile laturilor
(BC ), (AC ), respectiv, (AB ), iar h a , h b , h c sunt lungimile @n[l\imilor din A, din B ]i
respectiv, din C . Presupunem c[ sunt verificate inegalit[\ile (1) ]i c[ a  b  c. Se
deduce, f[r[ dificultate c[, @n acest caz, avem h a  h b  h c . Din asem[narea celor dou[
h h h
triunghiuri, deducem c[ au loc egalit[\ile: ca  cb  ac , (2). #n plus, avem ]i S 
a  ha b  hb c  hc
   , de unde rezult[: h a  2S 2S 2S
a ; hb  b ; hc  c .
2 2 2
Cu aceste egalit[\i, rela\iile (2), devine: ac  b  ac  2S 2S 2S
ac  b 2  ca , de unde
b
rezult[ b 2  ac sau b  ac , egalitatea care exprim[ faptul c[ num[rul b este media
geometric[ a numerelor a ]i c; evident, av`nd asemenea celor dou[ triunghiuri,
deducem c[ ]i @n triunghiul ale c[ror laturi au lungimile h a , h b , h c, are loc egalitatea
hb  ha  hc .
Aplica\ii:
1. Numerele a, b, c sunt lungimile laturilor unui triunghi. }tiind c[: ax  by  cz,
stabili\i, n ce condi\ii, numerele a, y, z pozitive, pot fi lungimile laturilor unui triunghi.
Solu\ie. Not[m cu k, cele tri produse egale, ]i astfel ob\inem: x  ak ; y  k ; z  kc .
b
Pentru a putea fi lungimile laturilor unui triunghi, trebuie s[ aib[ loc rela\iile:
 k  k  k  1  1  1
 a b c  a b c
 y  z  x  k k k  1 1 1
 z  x  y sau  b  c  a , adic[  b  c  a , de unde rezult[:
  k k k  1 1 1
     
 c a b  c a b
 bc  ac  ab

 ac  ab  ac , care exprim[ condi\iile de existen\[ ca un triunghi s[ aib[ lungimile x, y, z.
 cb  ac  bc

2. a) Ar[ta\i c[ exist[ un triunghi, ale c[rui @n[l\imi au lungimile egale cu 2 3 cm ]i 6
cm;
b) Determina\i lungimile laturilor acestui triunghi.
Solu\ie. a) Fie un triunghi ABC, av`nd AB  c, AC  b ]i BC  a ]i cu @n[l\imile
h a  2 3 cm, h b  3 2 cm ]i h a  6 cm. Din faptul c[ 2 3  3 2  6, deducem c[
a  b  c; au loc egalit[\ile:
ah a  bh b  ch c , adic[ 2a 3  3b 2  6c sau a  b  c  k, unde k  R .
1 1 1
2 3 3 2 6

Editura Cardinal 7
Astfel, avem: a  k ; b  k ]i c  k ; observ`nd c[ a 2  b 2  c 2 , deducem,
2 3 3 2 6
c[ exist[ triunghiul ABC, care este dreptunghi cu m(mA )  90 .
b) Pentru a determina lungimile laturilor, preciz[m c[ @n orice triunghi
dreptunghic, catetele sunt ]i @n[l\imi; a]adar, AB  h b  3 2 , AC  h c  6, iar
ipotenuza, adic[ latura (BC ), are lungimea egal[ cu (3 2 )  6 2  3 6 cm.
2

Preciz[m c[ triunghiul ABC este unic ]i se ob\ine c`nd k  a , deci k  6 6 .


1
2 3
3. a) Se d[ triunghiul ABC, cu BC  18cm, Ac  12cm, Ab  8cm. Ar[ta\i c[ se poate
construi un triunghi cu @n[l\imile triunghiului ABC.
b) Ar[ta\i c[ triunghiul construit cu @n[l\imile triunghiului ABC este asemenea cu
triunghiul ABC ]i determina\i raportul lor de asem[nare;
c) Ar[ta\i c[ dac[ dou[ triunghiuri sunt asemenea ]i laturile unui triunghi sunt
congruente cu @n[l\imile celuilalt triunghi, atunci @n fiecare din cele dou[ triunghiuri,
una dintre laturi este media geometric[ a celorlalte laturi.
Solu\ie. a) Calcul[m aria S a triunghiului ABC ]i avem
S  p(p  a )(p  b )(p  c )  9  7  11 .
ah b  hb c  hc
Din S  a   , unde h a , h b h c sunt lungimile @n[l\imilor din A, din B ]i
2 2 2
respectiv, din C @n triunghiul ABC; avem h a  2S 2S 2S
a ; h b  b ; h c  c . Prin @nlocuire se
arat[ c[ h a  h b  h c , h b  h a  h c , h c  h a  h b.
h h h
b) Avem: a  b  c  h a  h b  h c ; se verific[ u]or c[ ca  b  ac ,
b
adic[ laturile sunt propor\ionale ]i prin urmare triunghiurile sunt asemenea;
c) #n general, dac[ a, b, c sunt lungimile laturilor unui triunghi ]i h a , h b , h c
h h h
sunt lungimile @n[l\imilor, iar triunghiurile sunt asemenea cu ca  b  ac , atunci
b
2S 2S 2S
a  b  c , deci b 2  ac sau b  ac , adic[, num[rul b este media geo- metric[
c b a
a numerelor a ]i c. Din asem[narea triunghiurilor, deducem c[ are loc egalitatea:
hb  ha  hc .

Bibliografie:
1 Traian Lalescu, “Geometria triunghiului”, Editura Tineretului, Bucure]ti, 1958.
“Geometria triunghiului”, Editura Apollo, Craiova, 1993.

*
* *

8 Revista de matematic[ din Craiova


****************************************************************** ***
* CONCURSURI }I OLIMPIADE *
*********************************************************************

CONCURSUL DOLJEAN DE MATEMATIC{


19 martie 2016

Prezentare de: Prof. Dan Secl[man ]i Cristian Moan\[, Craiova


Prof. Mariana Ungureanu, Dr. Tr. Severin

A. ENUN|URI.

Clasa a IX-a
1. a) S[ se demonstreze c[ pentru n  1, este adev[rat[ inegalitatea: 3 n  n  2.
b) Determina\i func\ia f : N N av`nd proprietatea:
(m  3f (n) ) divide (f (m)  3n ), () m, n  N .
CRISTIAN MOAN|{

2. Fie ]irul de numere strict pozitive (x n ) n N , cu proprietatea:


x n (x n1  x n1 )  2x n1  x n1 , ( ) n  1.
Ar[ta\i c[: x 0  x 1 .
DAN SECL{MAN

3. Se consider[ ]irurile (a n ) n1 ]i (b n ) n1 cu a 1  1, astfel @nc`t:


5a n  3 a 3 ( )
a n1  ,b  n ,  n  1.
an  3 n an  1
a) Calcula\i a 2 , a 3 , b 1 , b 2 ]i b 3.
b) Ar[ta\i c[ (b n ) n1 este progresie geometric[.
c) Determina\i termenii generali ai celor dou[ ]iruri.
R.M.C. 1/2015-2016

4. Fie patrulaterul ABCD, H 1 ortocentrul triunghiului ABC ]i H 2 ortocentrul


triunghiului DBC. Demonstra\i c[ ABCD este inscriptibil dac[ ]i numai dac[
H 1 H 2  AD.
* * *

Editura Cardinal 9
Clasa a X-a
1. a) S[ se rezolve @n R ecua\ia: x log 24 14  x  2 log 24 x  x log50
24 .

b) Se consider[ numerele reale, a1 , a2 , . . ., a n  R , n  N, n  2 cu proprietatea


c[: a 1  a 2  . . .  a n  1. S[ se demonstreze c[:
log 2a 1 a 2 log 2a 2 a 3 log 2a n a 1
  ...   1.
na 1  (n  1 ) na 2  (n  1 ) na n  (n  1 )
CRISTIAN MOAN|{
2. a) Fie z 1 , z 2 , z 3  C astfel @nc`t z 1  z 2  z 3  0 ]i z 1  z 2  z 3  1.
Ar[ta\i c[: z  z 1 2  z  z 2 2  z  z 3 2  3(1  z 2 ), ( )z  C.
b) Deduce\i c[ dac[   CR este r[d[cin[ cubic[ a unit[\ii, atunci pentru orice
num[r complex z, are loc egalitatea:
z  1 2  z   2  z   2 2  3(1  z 2 ).
R.M.C. 1/2015-2016
lg x
3. a) Ar[ta\i c[ func\ia f : (0, 1 )  R, f (x )  x , este strict cresc[toare.
b) Determina\i x  0,  cu proprietatea: (sin x )  (cos x ) .
cos x sin x
2
* * *

4. Dac[ x, y  (1,  ) ]i log x (x  1 )  log y (y  1 )  0, ar[ta\i c[ x  y  4.


DAN SECL{MAN

Clasa a XI-a
1 3

1. Dac[ A  2 2 , determina\i n  N, n  20, cu proprietatea:
3 1
2 2
1 0
An  .
0 1
R.M.C. 1/2015-2016

2. Fie A  (a ij ) 1ijn , A  M n (R), n  4. Se construie]te matricea p[tratic[ de ordinul


n  1, B  (b ij ) 1i,jn , k  1, n, definit[ prin: b ij  a ij  a kk  a ik  a kj.
S[ se determine   C astfel @nc`t det B    det A.
CRISTIAN MOAN|{

10 Revista de matematic[ din Craiova


3. Se consider[ ]irul (a n ) n1 definit prin:
a  2a 2  . . .  na n a n1 ( )
a 1  1 ]i 1 n1  n ,  n  1.
an
S[ se calculeze: lim
n (n  1 )!
.
C{T{LIN CRISTEA

4. Fie f : RR, astfel @nc`t exist[   (0, 1 ), cu proprietatea:


f (x)    x , ()x R.
a) Rezolva\i @n R ecua\ia: f (x )  x.
b) Dac[ g n  f  f  . . .  f, n  N , ar[ta\i c[: lim g (2016 )  0.
n n
n ori

(f (x )) 2 x  (f (x ))
2
c) Calcula\i lim x ]i lim .
x0 x0 sin x
DAN SECL{MAN

Clasa a XII-a
1. Pe mul\imea numerelor reale se define]te legea de compozi\ie “ “ astfel:
x  y  xy  2x  2y  2, () x, y  R.
a) Demonstra\i c[ legea “ “ este asociativ[.
b) Determina\i a  R cu proprietatea: a  x  x  a  a, ( )x  R.
c) S[ se calculeze: 1  2  3  . . .  4032 .
2016 2016 2016 2016
* * *
2. Fie A, ,  un inel cu 0  1, care are cel pu\in 3 elemente av`nd proprietatea:
( )
ababa  a, ( ) a, b  A  0, 1. Demonstra\i c[:
a) Inelul (A, ,  ) nu are divizori ai lui zero.
b) Inelul (A, ,  ) este finit ]i determina\i num[rul de elemente ale inelului.
ANI DR{GHICI
3. Fie (a n ) n1 , (b n ) n1 dou[ \iruri cu termenii generali:

an  1  1  ...  1 , b  1  1 1 1
n1 n2 n  n n n n  3  n  8  . . .  n  n2  1 .

S[ se calculeze limitele celor dou[ ]iruri. Dac[ a lim a , b lim


n n
b , s[ se compare
n n
cele dou[ limite?
CRISTIAN MOAN|{

ln(x  1 )
3
4. Calcula\i integrala:  dx.
2 x2  1
* * *

Editura Cardinal 11
B. SOLU|II,
INDICA|II DE REZOLVARE, BAREME

Clasa a IX-a
1. a) Induc\ie matematic[. (2p)
b) Pentru m  n  1 ]i not`nd f (1)  k  N se ob\ine:
(1  3 k )  (k  3 ), deci 3 k  k  2. Dar 3 k  k  2, ( ) k  2  f (1 )  k  1. (2p)
Pentru m  1 ob\inem: 1  3 (  1  3  f n  n,  n  N  .
f (n) ) ( n) ( ) ( ) ( )
(1p)
Pentru n  1 ob\inem: (m  3)  (f (m)  3)  f (m)  m.() m  N ( ). (1p)
Din (  ), (  ) se ob\ine: f (n )  n, ( ) n  N . (1p)
2. Deoarece media armonic[ a dou[ numere pozitive este mai mic[ dec`t media
lor aritmetic[, din rela\ia dat[ @n ipotez[ ob\inem succesiv:
x  x n1 ( )
x n  n1 ,  n  1  x n  x n1  x n1  x n , ( )n  1. (2p)
2
Deci: x 1  x 2  x 0  x 1, x 2  x 3  x 1  x 2, .................., x n1  x n  x n  x n1. (1p)
Prin adunarea celor n inegalit[\i ob\inem:
x 1  x n  x 0  x n1 , ( )n  1  x 1  x 0  x n  x n1 , ( )n  1. (1p)
#n ultima rela\ie se dau valori lui n de la 1 la m, m  N ]i @nsum`nd cele m ine-
galit[\i se ob\ine: m(x 1  x 0 )  x 1  x m  x 1 , ( )m  1, unde s-a folosit faptul c[
xm  0, ()m  N. (1p)
Dac[ se presupune prin absurd c[ x 1  x 0 , din rela\ia anterioar[ se deduce c[ m 
x
 x 1 1 x 0 , ( )m  1, ceea ce este fals, deoarece mul\imea numerelor naturale nu este
m[rginit[ superior. A]adar: x 0  x 1 . (2p)
1
ALTFEL! Se noteaz[ x n  y n ]i rela\ia din ipotez[ devine:
y n1  y n1  2y n , ( )n  1.
Se procedeaz[ @n continuare ca @n solu\ia de mai sus.
3. a) a 2  51  33  2, a 3  10
23
 3  13 , b  1  3  1, b  2  3   1 , b  2   1 . (2p)
5 1
11 2
21 3 3 18 9
b n1 a n1  3 a n  1 2a n  6 a n  1 1,
b) Pentru orice n  1avem:     
bn a n1  1 a n  3 6a n  6 a n  3 3
ceea ce demonstreaz[ c[ ]irul (b n ) n1 este o progresie geometric[ cu ra\ia 1 . (2p)
n1
3
c) Conform punctului b), avem: b n   1 ,  n  1,  .
( ) ( ) (1p)
3
an  3 b 3 ( )
Din rela\ia dat[ @n ipotez[, b n  , ob\inem a n  n ,  n  1. (1p)
an  1 1  bn
|in`nd cont de rela\ia (  ) rezult[ imediat c[: a n  3n1 1 , ( )n  1.
n
(1p)
3 1

12 Revista de matematic[ din Craiova


4. Fie O 1 , O 2 centrele cercurilor circumscrise triunghiurilor ABC, DBC.
AH 1 BC ]i DH 2 BC; atunci: AH 1  DH 2. (1p)

|in`nd cont de aceasta: H 1 H 2  AD  AH 1 H 2 D paralelogram  AD 


 H1H2. (1p)
     
Avem: H 1 H 2  r H 2  r H 1 . (1p)
H 2 ortocentrul triunghiului BCD atunci:
   
O 2 H 2  O 2 B  O 2 C  O 2 D (Rela\ia lui Sylvester). (1p)

                
Avem: r H 2  r O2  r B  r O2  r C  r O2  r D  r H 2  r B  r C  r D  2r O2 .

     
Analog, se ob\ine:  r H 1  r B  r C  r A  2r O 1 . (2p)

          
Deci: H 1 H 2  AD  AD  r B  r C  r D  2r O 2  r B  r C  r A  2r O 1 

 
  
 AD  2 r O 2  r O 1  O 2  O 1  ABCD inscriptibil. (1p)

Clasa a X-a
1. a) Condi\ia de existen\[ x  0. Folosind formula a log b c  c log b a , ecua\ia se mai
scrie: x log 24 2log 24 7  x  x log 24 2  x log 24 2log 24 25  x log 24 2  x log 24 7  x  x log 24 2  x log 24 2 
x log 24 25  x log 24 2  (x log 24 7  x  x log 24 25 )  0. (1p)
Dar: x log 2  0  x log 7  x  x log 25  x log 7  x log 24  x log 25  7 log x  24 log x  25 log x .
24 24 24 24 24 24 24 24 24 (1p)
Cu substitu\ia log 24 x  t, t  R, ecua\ia se scrie: 7 t  24 t  25 t , ( ). (1p)
t  2 este solu\ia unic[ a ecua\iei (  ), iar x  576 este solu\ia ecua\iei din enun\. (1p)
b) Din a i  R , i  1, n ]i a 1  a 2  . . .  a n  1, deducem c[
a i  (0, 1 ), ()i  1, n, iar log a1 a 2 , log a2 a 3 , . . ., log an a 1  0. (1p)
2 2 2
log a 1 a 2 log a 3 a 3 log a n a 1
Not[m cu E    ...  ]i atunci:
na 1  n  1
( ) na 2  n  1
( ) na n  (n  1 )
E  12  n 2  E 
n 2 2 2
1 log a 1 a 2 log a 2 a 3 log a n a 1
 2    ...  
n na  (n  1 ) na  (n  1 ) na  (n  1 )
1 2 n

2 2 2
 na 1  (n  1 )  na 2  (n  1 )  ...  na n  (n  1 ) 
2 2
 12  log a 1 a 2  log a 2 a 3  . . .  log a n a 1  12  n  n log a 1 a 2  log a 2 a 3  . . .  log a n a 1 
n n
 12  n 2  1, unde pentru finalizarea exerci\iului din enun\ s-a folosit inegalitatea
n
Cauchy-Buniakovski-Schwarz ]i inegalitatea mediilor. (2p)
2. a) Not[m cu S membrul st`ng al egalit[\ii ce trebuie demonstrat[ ]i folosind
formula u  u  u 2 , ( )u  C, ob\inem:
S  (z  z 1 )( z  z 1 )  ( z 2  z 1 2  z  z 1  z 1  z ).

Editura Cardinal 13
Scriem suma desf[]urat[ ]i avem:
S  3  z 2  z 1 2  z 2 2  z 3 2  z(z 1  z 2  z 3 )  z (z 1  z 2  z 3 ). (  ). (2p)
Cum z 1  z 2  z 3  0, rezult[ z 1  z 2  z 3  0, adic[ z 1  z 2  z 3  0, (  ). (1p)
Din rela\iile (  ), (  ) ]i faptul c[ z 1 , z 2 , z 3 au modulele egale cu 1, se ob\ine imediat
concluzia cerut[. (1p)
b) Deoarece  este r[d[cin[ cubic[ a unit[\ii avem   1 ]i cum   CR, de-
3

ducem c[ 1     3  0. (1p)
Dar 1     2  1 ]i deci @n egalitatea de la punctul a) putem alege z 1  1, z 2  ,
z 3   2 ]i astfel punctul b) este demonstrat. (2p)

3. a) Fie a.b  (0, 1 ), astfel @nc`t a  b; atunci 1 1


a  b  0 ]i lg a  lg b  0, de unde
lg a lg a lg b
rezult[ a   , adic[ f este strict cresc[toare. (3p)
b b
b) Logaritm[m ambii membri ai ecua\iei @n baza zece ]i ob\inem f sin x  ( )
 f (cos x ), unde f este func\ia de la punctul a). (2p)
Deoarece f este strict cresc[toare, rezult[ c[ f este injectiv[ (1p)
]i atunci sin x  cos x ]i cum x  0,  , ob\inem x   . (1p)
2
2 4
4. Deoarece x  1  x , ( )x  1, cu egalitate numai pentru x  2, prin logaritmare
4
@n baza x, ob\inem: log x (x  1 )  2 1  1 , ( ), cu egalitate dac[ ]i numai dac[
log 2 x
x  2.
Analog, log y (y  1 )  2 1  1 , (  ), cu egalitate dac[ ]i numai dac[ y  2. (3p)
log 2 y
Prin adunarea rela\iilor (  ) ]i (  ), rezult[
log x (x  1 )  log y (y  1 )  2 2  1  1 , (1),
log 2 x log 2 y
cu egalitate dac[ ]i numai dac[ x  y  2. (1p)
|in`nd cont de ipotez[ ]i de (1), ob\inem: 1  1  2, (2), cu egalitate dac[ ]i
log 2 x log 2 y
numai dac[ x  y  2. (1p)
#n ipoteza x  1, y  1, avem log 2 x  0, log 2 y  0 ]i atunci:
1  1  4 , (3), cu egalitate dac[ ]i numai dac[ x  y.
log 2 x log 2 y log 2 x  log 2 y

(am \inut cont c[ (a  b ) 1 1


a  b  4, ( )a, b  0). (1p)
Din (2) ]i (3) rezult[ xy  4 cu egalitate numai pentru x  y  2. Cum x  y  2 xy ]i
xy  4, deducem c[ x  y  4, cu egalitate dac[ ]i numai dac[ x  y  2. (1p)

14 Revista de matematic[ din Craiova


Clasa a XI-a
cos x  sin x
1. Matricea dat[ o scriem sub forma A  , unde x   . Atunci
sin x cos x 3

cos x  sin x cos x  sin x cos 2x  sin 2x


A2    ]i prin induc\ie matema-
sin x cos x sin x cos x sin 2x cos 2x

cos nx  sin nx
tic[ se ob\ine: A n  , ( )n  N. (4p)
sin nx cos nx

1 0
Din condi\ia A n  ob\inem: cos n  1 ]i sin n  0. Deoarece n  20,
0 1 3 3
rezult[ n  0, 6, 12, 18. (3p)
2. Cazul 1: a kk  0.
a 11 a 12  a 1k  a 1n a 11  a kk a 12  a kk  a 1k  a 1n  a kk
a 21 a 22  a 2k a 2n a 21  a kk a 22  a kk  a 2k  a 2n  a kk
   1       
det A   . (2p)
a k1 a k2  a kk  a kn a n1
kk a k1  a kk a k2  a kk  a kk  a kn  a kk
      
a n1 a n2  a nk  a nn a n1  a kk a n2  a kk  a nk  a nn  a kk
Adun`nd la fiecare din coloanele de indice j  1, n, j  k coloanele de indice j @nmul\ite
cu (a kj ), ob\inem:
b 11 b 12  a 1k  b 1n
b 21 b 22  a 2k  b 2n
 
det A  1n1   (1 )  a kk  1n1  det B.
kk
a kk 0 0  a kk  0 a kk
 
b n1 b n2  a nk  b nn
Deci pentru a kk  0, det B  a n2 kk  det A, iar   a kk .
n2
(2p)
Cazul 2: a kk  0.
#n acest caz b ij  a ik  a kj , unde i, j  1, n, j  k, fiecare dou[ linii ale matricei B sunt
propor\ionale. Deci pentru a kk  0, det B  0  0  det A, iar   0. (2p)
#n concluzie det B  a kk  det A, iar   a kk .
n2 n2
(1p)

3. D[m valori lui n: n  1  a 2  1 ; n  2  a 3  4 . (1p)


2 3
(n  1 )  (n  1 )!
Demonstr[m prin induc\ie c[: a n  n , ( ) n  2.

Editura Cardinal 15
(k  1 )  (k  1 )! (k )  (k )!
P(2 ): a 2  1 , P(k ): a k  , P(k  1 ): a k1  . (3p)
2 k k1
#nlocuind @n rela\ia de recuren\[ n  k ob\inem:
a 1  2a 2  . . .  ka k a k1 1  1  . . .  (k  1 )(k  1 )! a k1
    a k1 
k1 k k1 k
k  1 k1 i  i!  k  1 k1 (i  1 )!  i!  k  k!.

k1 
i1 k1 
i1 k1
(2p)

Atunci: lim
an
lim n  1  1. (1p)
n (n  1 )! n n
4. a) Din ipotez[ rezult[ c[ f (0 )  0. Presupun`nd prin absurd c[ ar exista t  0
astfel ca f (t )  t, atunci @nlocuim x  t @n rela\ia din ipotez[ ]i ob\inem a  1, ceea ce
contrazice faptul c[ a  (0, 1 ). (2p)
b) Pentru x  f (x ) rela\ia dat[ @n enun\ devine:
f (f (x))  a  f (x)  a2  x , ()x  R. Continu[m ra\ionamentul ]i ob\inem:
f (f (f (x)))  a2  f (x)  a3  x , () x  R. (1p)
A]adar: g 2 (x )  a 2  x , g 3 (x )  a 3  x , ( )x  R. Prin induc\ie matematic[ rezult[
c[ pentru orice x  R avem: g n (x )  a n  x , ( )n  N . (1p)
Atunci g n (2016 )  2016  a n , ( )n  1 ]i cum lim n
a n
 0. (deoarece a  ( 0, 1 )),
rezult[ c[: lim
n n
g (2016 )  0. (1p)
f (x )
c) Din ipotez[ avem lim f (x )  0 ]i x  a, ( ) x  R 0; atunci prima
x0

limit[ cerut[ este @n cazul de nedeterminare 0 . Avem:


0
(f (x )) 2 f (x )
x  f (x )  x  a  f (x ) , ( )x  0,
(f (x )) 2
de unde rezult[: lim
x0 x  0. (1p)
x  (f (x ))
2 2
De asemenea avem: lim   lim x  1  (f (x ))  1. (1p)
x0 sin x x0 sin x x

Clasa a XII-a
1. a) Asociativitatea rezult[ imediat din calcul. (2p)
b) Pentru orice x  R avem: a  x  a  ax  2a  2x  2  a  a(1  x ) 
 2(1  x )  0  a  2.
Cum legea este comutativ[ avem ]i x  2  2 pentru orice x R. (3p)
c) |in`nd cont de asociativitatea legii, precum ]i de rezultatul punctului b),
ob\inem:
1  2  . . .  4031  4032  1  2  . . .  4031  2  . . .  2.(2p)
2016 2016 2016 2016 2016 2016 2016
2

16 Revista de matematic[ din Craiova


2. a) Presupunem c[ inelul (A, ,  ) ar avea divizori ai lui zero. (1p)
Rezult[ c[ exist[ a, b  A  0, 1 astfel @nc`t a  b  0. Atunci: ababa  a 
 (ab )(aba )  a  0  a, ( )a  A  0, 1, imposibil deoarece card A  3]i deci
inelul (A, ,  ) nu are divizori ai lui zero. (1p)
b) ( )a  A  0, 1  ababa  a  a(baba  1 )  a, a  0 ]i deci baba  1.
Dac[ lu[m a  b  a 4  1, ( )a  A  0, 1  a  a 3  a 3  a  1, ( )a  A  0, 1 
 elementul a este inversabil ]i deci: a 4  1  (a  1 )(a  1 )(a 2  1 ), ( ). (1p)
Au loc urm[toarele cazuri:
I. Dac[ 1  1, fie b  1@n rela\ia ini\ial[; rezult[ a 3  a  a(a 2  1 )  0  a 2 
 1, ( )a  A  0  (a  1 )(a  1 )  0 ]i cum a  1  0  a  1. #n acest caz A 
 0, 1, 1 este un corp cu trei elemente. (2p)
II. Dac[ 1  1  1  1  0, iar din (  ) deducem c[: a 2  1 sau a 2  1 
 (a  1 )(a  1 )  0  (a  1 ) 2  0  a  1, fals! Deci inelul (A, ,  ) este corp comu-
tativ cu trei elemente. (2p)
3. Transform[m ]irul (a n ) n1 , astfel:
an  1  1  . . .  n  1  1 1 1
n n 1  1  1  2  . . .  1  nn .
1
n1 n2 n n

Vom considera func\ia f : 0, 1  R, f (x )  1 , ]i pentru fiecare n  1, form[m


x1
diviziunea echidistant[  n a intervalului 0, 1 de forma:

n  x0  0  x1  1 1 n
n  . . .  xk  k  . . .  xn  n  1 ,
de norm[  n   1 n , iar @n fiecare interval x k1 , x k  se alege punctul intermediar de
1
forma:  k  n . Suma Riemann asociat[ func\iei f, diviziunii  n ]i punctelor intermedi-
are  k este:
( n ,  )  f ( k )  (x k  x k1 )  1 k  nk  k 
n n
1 n 1 a .
k1 k1 1  n
n 
k1 n  k
n (2p)
Rezult[
1 1
lim 1  1  ...  1
n  n lim  (f,  )  f (x )dx  ln(1  x )  ln 2  a.(1p)
n n1 n2 n  n
0 0

Transform[m ]irul (b n ) n1 , astfel:

bn  1 1 1
n  n  3  n  8  ... 
1  1 1
n 10 
1  ...  1 .
n  n2  1 3 n 2 1
1 n 1 n

Vom considera deci func\ia f : 0, 1  R, f (x )  1 , ]i pentru fiecare n  1, form[m


x1
diviziunea echidistant[  n a intervalului 0, 1 de forma:

 n  x 0  0  x 1  1 1 n
n  . . .  xk  k  . . .  xn  n  1 ,
de norm[  n   1
n , iar @n fiecare interval x k1 , x k  se alege punctul intermediar de

Editura Cardinal 17
k2  1
forma:  k  n . Suma Riemann asociat[ func\iei f, diviziunii  n ]i punctelor
intermediare  k este:
 nk  k n 1  
n n n
( n ,  )   f ( k )  (x k  x k1 )   1 1  b n . (2p)
k1 k1 k2  1 k1 n  k2  1
1 n

Rezult[ lim 1 1  1  ...  1 lim   (f,  ) 


n n n  n
n 3 n 8 n  n2  1
1 1
 f (x )dx  ln(1  x )  ln 2  b. (1p)
0 0
Deci a  b. (1p)

4. Not[m x  t  1 ]i rezult[ c[ dx   2 2 dt. (2p)


t1 (t  1 )

ln t  1  1 3 ln
2
1 dt  ln 2  ln(t  1 ) dt 
2 3
Atunci: l  t  1  2 dt  t  (2p)
3 t  1 2  1 (t  1 ) 2 2 t2  1 2 t2  1
t1
ln(t  1 )
3 3 3
 t 2ln21 dt  2  1 dt  ln 2  t 2 1 1 dt  l. (1p)
2 2 t 2

3 arctg 1
Dar  1 1 ]i ob\inem: l  7 ln 2.
t 2  1 dt  arctg 3  arctg 2  arctg 7
(2p)
2 2

*
* *

18 Revista de matematic[ din Craiova


************************************************************ *********
* PROBLEME PROPUSE *
*********************************************************************

CICLUL PRIMAR
Clasa I
I.1. Dintr-un num[r mai mic dec`t 24 scad 18. Ce rest pot ob\ine?
I.2. Ia dintr-un num[r scris cu dou[ cifre identice diferen\a numerelor 90 ]i 20.
I.3. Scrie-l pe 20 ca o sum[ de 4 termeni egali.
I.4. Dac[ sc[dem 2 la vecinii unui num[r ob\inem 14 ]i 16. Care este num[rul?
I.5. G[se]te toate numerele n naturale formate din zeci ]i unit[\i la care num[rul
zeceste cu 2 mai mare dec`t cel al unit[\ilor.
I.6. Calculeaz[ num[rul necunoscut:
42  6    5 34      37
7  5    94 76      72
  2  3  39 2  45    44
88    3  82   3  2  61
I.7. Calcula\i valoarea lui a ]i b din egalit[\ile:
 a  3  9  a  a  b  b  24
 b  a  2 ;  ab2
.
 
I.8. G[si\i dou[ numere egale a c[ror sum[ sa fie cu 5 mai mare dec`t diferen\a
numerelor 86 ]i 51.
I.9. Afl[ suma dintre 10 ]i cel mai mare num[r de o cifr[.
I.10. Scrie fiecare num[r la locul potrivit:

17-11 20-12 14+5 20-19 13+4 15+3

0 5 10 15 20

I.11. Adun[m trei numere consecutive, ultimul s[ fie 7. Ce num[r ob\inem?


I.12. Din cei 67 lei pe care @i avea Sorin, @]i cump[r[ 2 caiete identice, o carte care
cost[ 21 lei ]i prime]te rest 16 lei. C`t cost[ un caiet?
I.13. Dac[ sora mea @mi d[ jum[tate din banii ei, mai pun 23 lei ]i pot cump[ra 2
mingi a 34 lei. C`\i lei are sora mea?

Editura Cardinal 19
I.14. Ca s[ cumpere un pix care cost[ 90 de lei, la suma pe care o avea, Maria a
mai ad[ugat o bancnot[ de 10 lei ]i dou[ monede de 5 lei.
I.15. Din cel mai mare num[r format numai din zeci sc[de\i dublul sumei dintre cel
mai mic num[r format din zeci ]i unit[\i ]i cel mai mare num[r format numai din
unit[\i.
I.16. Ai cartona]ele: 0 2 1 3.
Afl[ cele dou[ numere formate din zeci ]i unit[\i:
a) a c[ror sum[ s[ fie cea mai mare;
b) a c[ror diferen\a s[ fie cea mai mare;
c) a c[ror sum[ s[ fie cea mai mic[;
d) a c[ror diferen\[ s[ fie cea mai mic[.
I.17. Dac[ dintr-o l[di\[ se iau 2 mere ]i se pun @n alt[ l[di\[, atunci @n fiecare
l[di\[ r[m`n c`te 35 de mere. C`te mere au fost la @nceput @n fiecare l[di\[?
I.18. M[ g`ndesc la un num[r, @l adun cu 30 ]i ob\in 70. La ce num[r m-am g[ndit?
I.19. Alin a dat prietenului s[u 12 castane ]i i-au mai r[mas tot at`tea castane c`t
r[sturnatul num[rului 6. C`te castane a avut Alin?
I.20. Afla\i suma numerelor x, y ]i z, dac[: x  12  1, y  30  11, z  69  27.
I.21. Scrie: a) cel mai mic num[r natural scris cu dou[ cifre diferite;
b) cel mai mare num[r natural scris cu dou[ cifre diferite;
c) cel mai mare num[r natural scris cu dou[ cifre identice;
d) cel mai mic num[r natural scris cu dou[ cifre consecutive.
I.22. Scrie toate numerele naturale de dou[ cifre care au:
a) cifra zecilor cu 3 mai mare dec`t cifra unit[\ilor;
b) cifra unit[\ilor cu 1 mai mic[ dec`t cea a zecilor.
I.23. C`te piersici a avut bunica dac[ fiecare dintre cei trei nepo\i ai s[i a primit 5
piersici, iar ei i-au mai r[mas tot at`tea?
I.24. D[nu\ se uit[ @n fiecare zi c`te 10 minute la televizor, iar s`mb[ta ]i duminica
c`te 15 minute pe zi. C`te minute se uit[ D[nu\ @ntr-o s[pt[m`n[?
I.25. #n urm[ cu 3 ani, Irina, Crina ]i Cristina aveau @mpreun[ 20 de ani. C`\i au
@mpreun[ anul acesta?

Clasa a II-a
II.1. #ntr-o clas[ sunt 29 elevi, dintre care 17 nu sunt fete. Cu c`t este mai mare
num[rul b[ie\ilor dec`t al fetelor?
II.2. #ntr-o cutie sunt 67 bile verzi, galbene ]i albe, 39 bile nu sunt galbene, 44 nu
sunt albe. C`te bile sunt de fiecare culoare?
II.3. M[ g`ndesc la un num[r a. Adaug r[sturnatul num[rului 82 m[rit cu 13. La

20 Revista de matematic[ din Craiova


rezultatul nou ob\inut adaug diferen\a dintre cel mai mare num[r par scris cu dou[
cifre ]i num[rul 75. Ob\in rezultatul 83. La ce num[r m-am g`ndit?
II.4. #ntr-un bidon @n care mai era ap[ s-au pus 18 litri, apoi s-au luat 7 litri. C`\i
litri de ap[ au fost la @nceput, dac[ acum sunt @n vas 45 de litri?
II.5. Desc[zutul este 69, sc[z[torul este cel pu\in egal cu 65, s[ se afle restul sau
diferen\a.
II.6. Dac[ restul este cel mult egal cu 5, iar sc[z[torul este 15, s[ se afle desc[zutul.
II.7. Afla\i valoarea necunoscutei din egalit[\ile:
39  a 14  12  2  31, a 54  16  24  7  35.
II.8. Dac[ mic]orez un num[r cu vecinul mic impar al num[rului 12, rezultatul @l
m[resc cu dublul num[rului 12 ]i noul rezultat @l mic]orez de dou[ ori cu num[rul 19,
ob\in num[rul 11. La ce num[r m-am g`ndit?
II.9. Fiul are 18 ani, iar tat[l are 44 de ani. Afla\i cu c`\i ani @n urm[ suma v`rstelor
lor era egal[ cu 54.
II.10. Mirela ]i-a propus s[ str`ng[ 72 de castane. #n prima zi ea str`nge 14
castane, iar @n fiecare din zilele urm[toare cu 10 castane mai mult dec`t @n ziua prece-
dent[. #n c`te zile termin[ Mirela de str`ns castanele?
II.11. Andrei are 43 de timbre, iar Marius are 31 de timbre. C`te timbre trebuie s[-i
dea Andrei lui Marius, pentru ca am`ndoi s[ aib[ acela]i num[r de timbre.
II.12. Suma a 3 numere naturale este 21. Care pot fi numerele, dac[ sunt @ndeplin-
ite, pe r`nd, urm[toarele condi\ii:
a) unul din termeni este de forma aa;
b) un termen este un num[r de forma ab, format cu ajutorul cifrelor 6 ]i 1;
c) a  a  c  21; unde a  c;
d) a  a  a  21. Ce observi?!
II.13. Dac[ Mihai ar avea cu 5 ani mai mul\i ar fi cu 20 de ani mai mic dec`t mama
sa, care este cu 2 ani mai t`n[r[ dec`t tat[l lui Mihai.
a) Care este suma v`rstelor p[rin\ilor lui Mihai c`nd acesta s-a n[scut?
b) Dac[ Mihai are 8 ani, care este suma v`rstelor lor?
II.14. Din cei 90 de lei pe care-i are, Alina a cump[rat dou[ caiete a c`te 3 lei
fiecare, a dat 30 de lei pe o pereche de pantofi ]i 15 lei pe o carte.
C`\i lei mai are Alina? (Rezolva\i @n dou[ moduri).
II.15. Dup[ ce ]i-a dublat suma de bani pe care o avea, Dorin a cump[rat cu 14 lei
o minge ]i cu 34 lei o ma]inu\[. Lui i-au mai r[mas 40 de lei. Ce sum[ ini\ial[ (la
@nceput) a avut?
II.16. Alina, Barbu ]i Costinel au @mpreun[ 80 de lei. Alina ]i Barbu au @mpreun[
40 de lei, iar Barbu ]i Costinel 61 de lei. C`\i lei are fiecare dintre ei?
II.17. Anghel are 60 de ghinde, iar Georgiana 42. C`te ghinde au @mpreun[? C`te
ghinde ar trebui s[ primeasc[ Georgiana de la Anghel pentru a avea acela]i num[r de
ghinde?

Editura Cardinal 21
II.18. G[se]te posibilele valori ale literelor: a, b, c, d, e!
a  a  9; b  b  36; c  c  16; d  d  49; e  e  25.
II.19. Adev[rat sau fals (A/F)?
a) 8 este jum[tatea lui 20! b) 5 este sfertul lui 20!
c) 4 este dublul lui 2! d) Jum[tatea lui 10 este 5!
II.20. Maria a cules din gr[din[ 27 de lalele. Ea i-a oferit Arianei o treime din
lalelele culese. C`te lalele a primit Ariana?
II.21. La un concurs de matematic[, Ana a ob\inut un num[r dublu de puncte fa\[
de Ina. Dac[ Ina ar mai fi primit 86 de puncte, ele ar fi avut acela]i num[r de puncte.
C`te puncte a ob\inut fiecare?
II.22. C`te buline con\ine un joc, ]tiind c[ cele 177 de buline ro]ii sunt cu 49 mai
pu\ine dec`t num[rul bulinelor albastre ]i cu 53 mai multe dec`t num[rul bulinelor
galbene?
II.23. O carte de pove]ti are 90 de pagini. Maria cite]te @n prima zi 13 pagini, @n a
doua zi un num[r dublu de pagini fa\[ de prima zi, @n a treia zi c`t r[sturnatul
num[rului de pagini din prima zi.
C`te pagini mai are de citit Maria pentru a termina cartea?
II.24. La un concurs de matematic[ se acord[ 15 puncte pentru fiecare problem[
rezolvat[ corect ]i 7 puncte pentru fiecare exerci\iu rezolvat corect. C`te puncte a
acumulat un elev care a rezolvat corect 3 probleme ]i 2 exerci\ii?
II.25. #ntr-o vaz[ sunt 35 de flori, lalele ]i narcise. Dac[ se iau 3 lalele ]i 2 narcise,
@n vaz[ r[m`n tot at`tea lalele c`t ]i narcise. C`te flori din fiecare fel sunt @n vaz[?

Clasa a III-a
III.1. #n depozit sunt 163 mingi albe, ro]ii ]i verzi. Mingi albe ]i ro]ii sunt 140, iar
ro]ii ]i verzi sunt 72. C`te mingi sunt din fiecare?
III.2. La flor[rie sunt bujori, trandafiri ]i garoafe. Num[rul trandafirilor este de 3
ori mai mare dec`t num[rul bujorilor, iar al bujorilor este de 6 ori mai mic dec`t
num[rul garoafelor. }tiind c[ num[rul garoafelor este @mp[tritul lui 12, afl[ c`te flori
sunt @n total?
III.3. Afl[ un num[r natural, ]tiind c[ sfertul lui se m[re]te de 2 ori, iar rezultatul
se m[re]te cu 16 ]i se ob\ine dublul celui mai mic num[r natural de dou[ cifre care are
suma cifrelor 5.
III.4. #ntr-o livad[ sunt nu mai pu\in de 50 ]i nu mai mult de 59 pomi. C`\i pomi
de fiecare fel sunt, dac[ o treime dintre ei sunt meri, jum[tate sunt peri, iar restul cai]i?
III.5. Din cel mai mare num[r impar de o cifr[ scad un num[r “a”, apoi adun
dublul num[rului 24 ]i ob\in un num[r mai mare dec`t jum[tatea num[rului 100.

22 Revista de matematic[ din Craiova


III.6. Monica are un num[r triplu de castane fa\[ de Onica. C`te castane are fiecare
dac[ Onic[i i-ar mai trebui 20 de castane astfel @nc`t fiecare s[ aib[ acela\i num[r de
castane?
III.7. Scrie\i un num[r natural de trei cifre, dac[ suma cifrelor sale este 21, suma
cifrelor ce reprezint[ sutele ]i unit[\ile este 16, iar suma cifrelor ce reprezint[ sutele ]i
zecile este 14.
III.8. G[si\i trei numere naturale impare distincte care @ndeplinesc simultan
urm[toarele condi\ii: I. ab  bc  ac  71. II. bc  ab  20.
III.9. S[ se determine toate numerele naturale care @mp[r\ite la acela]i num[r “n”
dau c`tul 11, iar dac[ se adun[ resturile se ob\ine suma 15.
III.10. Fie x  y  z numere pare consecutive. Care este valoarea fiec[ruia ]tiind:
12  x  y  z  24.
III.11. Cu banii pe care @i are, Elena poate cump[ra 7 stilouri ]i 9 pixuri. Se
r[zg`nde]te ]i cump[r[ de to\i banii numai pixuri pentru a oferi colegilor de clas[ c`te
1 pix. C`te pixuri cump[r[ ]i c`\i elevi sunt @n clasa Elenei dac[ un stilou cost[ c`t
dou[ pixuri?
III.12. Adrian are de 3 ori v`rsta lui Mircea. Peste 5 ani v`rsta lui Adrian va fi
dublul v`rstei lui Mircea. C`\i ani are acum fiecare dintre cei doi copii?
III.13. #n clasa noastr[ suntem 24 de elevi. Un sfert din num[rul total @l reprezint[
fetele. #n clasa prieten[ sunt de dou[ ori mai pu\ini b[ie\i ]i de dou[ ori mai multe fete
dec`t @n clasa noastr[. #ntr-o pauz[ ne juc[m @mpreun[ organiz`ndu-ne @n echipe de
c`te 9 membri. C`te echipe s-au putut forma?
III.14. Radu @]i num[r[ ghindele cu care se joac[. El observ[ c[ atunci c`nd le
grupeaz[ c`te dou[ ob\inute cu 8 mai multe gr[mezi dec`t atunci c`nd le num[r[ c`te
3. C`te ghinde are Radu?
III.15. Un antrenor sportiv @mparte c`te 3 mingi de oin[ ]i de dou[ ori mai multe
mingi de baschet la fiecare 8 elevi care particip[ la o activitate sportiv[. }tiind c[ s-au
folosit 63 de mingi, afla\i c`\i elevi au participat la acea activitate.
III.16. Andrei ofer[ fiec[rui coleg c`te un m[r din merele culese de el ]i @i mai
r[m`n 8 mere. Dac[ ar mai fi avut 2 colegi num[rul merelor oferite ar fi reprezentat
trei sferturi din num[rul merelor culese. C`\i elevi sunt @n clasa lui Andrei?
III.17. Afla\i num[rul natural “a” din egalitatea a  a  a  a  18.
III.18. Calcula\i rapid: 6  5  5  5  5  5  5  5  5  4 
141141141141141
III.19. Elena a @ntrebat-o pe Alina:
- De ce n-ai venit la ziua bunicii? Ieri ea ]i-a s[rb[torit a cincisprezecea oar[ ziua
de na]tere. Spune\i c`\i ani are bunica ]i @n ce zi a avut loc discu\ia?
III.20. Determina\i cifra a dac[: aa  2a  a9.
III.21. Ob\ine\i num[rul 14, folosind cinci cifre de 2, opera\ii aritmetice ]i paran-
teze.

Editura Cardinal 23
III.22. Afla\i un num[r de trei cifre abc, ]tiind c[ suma cifrelor este 23. Suma
primelor dou[ cifre este 17, iar diferen\a ultimelor dou[ este 2.
III.23. Determina\i numerele de dou[ cifre care au suma cifrelor egal[ cu cifra
zecilor.
III.24. Suma a trei numere naturale este 738. Dac[ dubl[m primul num[r @l
ob\inem pe al doilea, iar dac[ din al treilea num[r @l sc[dem pe al doilea, ob\inem
primul num[r. Care sunt cele trei numere?
III.25. #ntr-o clas[ sunt 25 de elevi. Din c`te familii provin elevii din acea clas[
dac[ exist[ dou[ grupe de gemeni?

Clasa a IV-a
IV.1. #n anul 2006, suma v`rstelor mamei ]i fiicei sale este 25 de ani. #n anul 2010,
v`rsta mamei va fi dublul v`rstei fiicei. C`\i ani a avut fiecare @n anul 2005?
IV.2. Doi colegi, Mircea ]i Vlad, au de aflat restul @mp[r\irii 23000 : 6000. Mircea
a r[spuns c[ restul este 5 proced`nd astfel: 23000 : 6000  3(rest 5 ), iar Vlad a indicat
alt rest. Cine are dreptate? Justifica\i!
IV.3. G[se]te numerele naturale care @mp[r\ite la 125 dau restul egal cu cubul
c`tului.
IV.4. Pentru a descifra un mesaj, un detectiv trebuie s[ ordoneze literele @n ordinea
descresc[toare a codurilor lor: P este o treime din S, I este cu 500 mai mic dec`t P ]i
dublul lui O, iar N este un sfert din I, adic[ 500. Care este mesajul?
IV.5. #ntr-o parcare sunt cu 7 mai multe ma]ini albe dec`t ro]ii. Vin 4 ma]ini albe
]i pleac[ 3 ma]ini ro]ii, num[rul ma]inilor ro]ii devenind a treia parte din num[rul
celor albe. C`te ma]ini albe ]i ro]ii au fost la @nceput @n parcare?
IV.6. La o serbare ]colar[ au participat, @n afara cadrelor didactice, p[rin\i ]i copii,
@n total 123 persoane. Se precizeaz[ c[ cei prezen\i au fost: cu 11 mame mai multe
dec`t ta\i, 26 de copii mai mult dec`t mame; dintre copii cu 18 fete mai multe dec`t
b[ie\i. Se cere s[ afla\i num[rul celor prezen\i pe categorii: mame, ta\i, b[ie\i, fete.
IV.7. Afla\i numerele a, b, c, d ]tiind c[ verific[ simultan rela\iile:
a0a  b2b  c3c  ddd.
2  d  20  10  5  14  18  (3  2  6  2 )  5  12.
IV.8. Ion ]i Mihai, afla\i la 300 m distan\[, se @ndreapt[ unul spre cel[lalt, Ion
merg`nd cu 7, 2 km/h, iar Mihai cu 3, 6 km/h. C[\elul lor alearg[ de la Ion spre Mihai,
apo @napoi la Ion ]i iar @napoi la Mihai ]i tot a]a p`n[ ce Ion ]i Mihai se @nt`lnesc.
Dac[ viteza lui este de 18 km/h, afla\i ce distan\[ a parcurs acesta.
IV.9. Bunicul are v`rsta c`t fiul ]i tat[l @mpreun[. C`nd s-a n[scut tat[l, bunicul
avea v`rsta pe care o avea tat[l c`nd s-a n[scut fiul. Ce v`rst[ are fiecare dac[ acum 10
ani tat[l avea de 3 ori v`rsta fiului?

24 Revista de matematic[ din Craiova


IV.10. Un num[r este cu 2006 mai mare dec`t cel[lalt; @mp[r\ind num[rul mai
mare la num[rul cel mic ob\inem c`tul 1 ]i un rest. Afla\i restul.
IV.11. Afla\i valoarea numeric[ a literei x din expresia:
4  8  12  16  . . .  440  4  (x  105 ).
IV.12. #ntr-o clas[, pe fiecare banc[ este un m[r. Dac[ fiecare elev din clas[
man`nc[ o jum[tate de m[r, r[m`n 5 mere. Dac[ stau c`te 3 elevi @n banc[, r[m`n 10
b[nci libere. Afla\i num[rul merelor, elevilor ]i b[ncilor din acea clas[.
IV.13. Alina, Bianca, Mimi ]i Roxana au aceea]i sum[ de bani. Dup[ ce Alina
cheltuie]te 3 lei pentru un bilet la loto ]i astfel @]i dubleaz[ suma c`]tig`nd, ea are c`t
celelalte trei @mpreun[ dup[ ce au pierdut (@mpreun[) de patru ori 4 lei la Jack Pot.
C`\i lei a avut fiecare la @nceput?
IV.14. Care numere naturale de trei cifre, cu cifrele a, b, c diferite de 0, satisfac
egalitatea: abc  acb  bac  bca  cab  cba  xxx.
IV.15. Dac[ 2 veveri\e m[n`nc[ @n 3 zile 60 de ghinde, atunci 5 veveri\e @n c`te
zile m[n`nc[ 100 de ghinde, ]tiind c[ zilnic fiecare veveri\[ consum[ acela]i num[r de
ghinde?
IV.16. Miaunic[, Mot[nic[ ]i Miorl[ic[ sunt 3 pisoi. Ta\ii lor au acelea]i nume,
astfel @nc`t nici unul dintre ei nu are numele identic cu cel al pisoiului s[u. }tiind c[
Miaunic[ nu este tat[l lui Miorl[ic[, atunci el este tat[l lui . . . . . ., Mot[nic[ este tat[l
lui . . . . . ., iar Miorl[ic[ este tat[l lui . . . . . . .
IV.17. Suma a 3 numere este 150. Primul num[r este c`t dou[ treimi din suma
ultimelor dou[ numere, iar diferen\a dintre al treilea ]i al doilea num[r este c`t un sfert
din al doilea num[r. Care sunt numerele?
IV.18. O feti\[ are acum 10 ani, iar mama sa 30 de ani. Afla\i peste c`\i ani feti\a
va avea v`rsta de dou[ ori mai mic[ dec`t mama sa.
IV.19. #n gr[dina zoologic[ sunt: lei, tigri ]i ]erpi. Ei au @n total 476 picioare ]i 131
capete, iar tigrii sunt cu 50 mai mul\i dec`t ]erpi. C`te animale de fiecare fel sunt?
IV.20. Afla\i 3 numere, ]tiind c[ produsul primelor dou[ este 84, produsul ultime-
lor dou[ este 252, iar suma dintre primul ]i ultimul este 12.
IV.21. Un casier fiind @ntrebat c`t a @ncasat @ntr-o zi, a r[spuns:
- Dac[ a] mai fi @ncasat @nc[ un sfert din c`t am @ncasat ]i @nc[ 500 lei, atunci a] fi
@ncasat 5500 lei. C`t a @ncasat casierul @n acea zi?
IV.22. S[ se @mpart[ num[rul 40 @n trei p[r\i, astfel @nc`t prima parte @mp[r\it[ la 2,
a doua @mp[r\it[ la 3 ]i a treia @mp[r\it[ la 5, s[ dea acela]i c`t.
IV.23. Frac\ia 3x  8 este echiunitar[ pentru x  . . . . . .
453
IV.24. S[ se afle un num[r de 4 cifre, ]tiind c[ dac[ @i a]ez[m @n fa\[ cifra 8,
ob\inem un num[r de trei ori mai mare dec`t num[rul ce s-ar ob\ine din num[rul ini\ial
cu cifra 6 a]ezat[ la sf`r]it.
IV.25. Cu 2 ani @n urm[, v`rsta mea era a ]asea parte din v`rsta mamei. Peste 3 ani
v`rsta mea va fi cu 6 ani mai mic[ dec`t jum[tate din v`rsta de acum a mamei.
C`\i ani am eu? Dar mama?

Editura Cardinal 25
Au colaborat la realizarea acestui num[r:
DORINA BR#NDU}E - }c. Gen. Nr. 3, Cugir, DANIELA DR{GOESCU - }c. Gen.
“G. Usc[tescu”, Tg. C[rbune]ti, CAMELIA BULIGIU - }c. Gen. “G. Usc[tescu”, Tg.
C[rbune]ti, MARIANA C#L|ARU - }c. Gen. “G. Usc[tescu”, Tg. C[rbune]ti,
LOREDANA ELENA PETRIC{ - }c. Gen. “G. Usc[tescu”, Tg. C[rbune]ti, DORIN
BR#NDU}E - }c. Gen. Nr. 3, Cugir, NICOLETA CHI|ORAN - Rovinari, Gorj, RODICA
CRAU - }c. Gen. “G. Usc[tescu”, Tg. C[rbune]ti, SILVIA AVRAMESCU - }c. Gen. “G.
Usc[tescu”, Tg. C[rbune]ti, GHERGHINA LUNGEANU - Gala\i, SIMONA-ELIZA
MAZILU - Rovinari, Gorj, ADRIANA C#RSTEA - Tg. C[rbune]ti, LILIANA
ARGENTOIANU - C. N. “Fra\ii Buze]ti”, Craiova, ELENA CIOLACU - }c. Gen.
"Grigore Geam[nu", Turcine]ti, Gorj, GABRIELA MATI} - }c. Gen. Nr. 26, Timi]oara,
DORINA BU}OI - }c. Gen. Bor[scu, Gorj, ANDA TEC}A - }c. cu cls. I-VIII Nr. 3, Cugir,
CORINA ELENA BIANCHI - }c. Gen. “Arcani”, Arcani, ELENA CRI}AN - }c. Gen. Nr.
3 Cugir, DANIELA ANDREESCU - }c. Gen. "Grigore Geam[nu", Turcine]ti, Gorj,
LILIANA VASILESCU - }c. Gen. “G. Usc[tescu”, Tg. C[rbune]ti, ELENA POPESCU -
Lic. cu Prog. Sportiv, Tg. Jiu, ELENA NICOLAE - }c. Gen. Ple]oi, Dolj,
IONELIA-SIMONA NAE - Booveni, Dolj, MARIANA Z{LOG - Tg. Jiu, SIMONA
CROITORU - Gureni, Dolj, IRINA BARBU - Craiova, IOANA PI|A - Vi]ina, Olt,
AUREL }TEFAN - C. N. “Elena Cuza”, Craiova, MARIAN CIUPERCEANU - C. N.
“Fra\ii Buze]ti”, Craiova, LETI|IA DR{GHICI - C. N. “Fra\ii Buze]ti”, Craiova,
VERONICA PRUFU - C. N. “Fra\ii Buze]ti”, Craiova, CRISANDA-GEORGIA TURCU -
C. N. “Fra\ii Buze]ti”, Craiova, IONELA BARB{RAS{ - C. N. “Fra\ii Buze]ti”, Craiova,
BADEA ADELA - Craiova, VILIANA CONSTANTIN - Craiova, LAURA ZAHARIA -
Craiova, MIRELA SIMONA POPESCU - C. N. “Elena Cuza”, Craiova, ROXANA
POPESCU - Craiova, LAVINIA VULPE - Craiova, DIANA BORANGIC - Craiova,
LILIANA GENTOIANU - Craiova, EUGENIA PETCU - }c. Gim. "Gh. |i\eica", Craiova,
PETCU VIORICA ANGELICA - }c. Primar[ "Vasile Carabis", C@mpofeni, Gorj, IONICA
TRIC{ - }c. Primar[ P[i]ani, Stoina, Gorj, RODICA CRAU - }c. Gen. "George Usc[tescu",
Tg. C[rbune]ti, ALINA VL{DU|ESCU - }c. Gim. "George Usc[tescu", Tg. C[rbune]ti,
VALENTINA-IRINA COVERC{ - }c. Gim. Ureche]ti, Ureche]ti, Gorj, ANA PAN{ - }c.
Gen., Cru]e\, Gorj, GABRIELA MATI} - }c. Gim. Nr. 26, Timi]oara, ECATERINA
BOGDAN - C. N. "Carol I", Craiova, LICA IONEL OVIDIU - C. N. "Carol I", Craiova,
MARIA DORAN - C. N. "Carol I", Craiova, ION ROTARU - C. N. "Carol I", Craiova,
COCA TANCIU - C. N. "Carol I", Craiova, CATARGENA RADA - C. N. "Carol I",
Craiova, MIHAELA LIC{ - C. N. "Elena Cuza", Craiova, DINU CONSTANTINA - }c. cu
clasele I-VIII, Castranova, CORINA MIRELA ENEA -C. N. "Carol I", Craiova, LIANA
VELI}CU - C. N. "Carol I", Craiova, ISABELLA NICA - C. N. "Carol I", Craiova, IOANA
PI|A - Vi]ina, Olt, ELENA GRIGORE - C. N. "Fra\ii Buze]ti", Craiova, LAURA
ZAHARIA - C. N. “Elena Cuza”, Craiova, ELIZA CHILOM - "}c. cu cls. I-VIII", Murga]i,
SIMONA CECILIA TUDORAN - C. N. "Carol I", Craiova, DENISA BISTRICEANU - C.
N. "Carol I", Craiova, VERONICA COTORA - C. N. "Carol I", Craiova, FLORENTINA
ZULEANU - }c. Nr. 9 "Petrache Poenaru", Craiova, VIRGINIA DOBRESCU - "}c. cu
cls. I-VIII", Desa, IRINA NUT{ - C. N. "}tefan Velovan", Craiova, MIHAELA RISTEA -
C. N. "Carol I", Craiova, IRINA PETRE - C. N. "Carol I", Craiova, GEORGIANA TOMA
- C. N. "Carol I", Craiova, TUDOR C{LIN - "}c. cu cls. I-VIII", D[buleni, ALEXANDRA
C{LIN - Lic. Teoretic "Constantin Br`ncoveanu", D[buleni, ADELA PAULA C{LIN -
Gr[dini\a Nr. 17, Craiova, FLORENTINA POPA - C. N. "Carol I", Craiova, LIANA
VELISCU - C. N. "Carol I", Craiova, GETA TEAU - C. N. "Carol I", Craiova, LIVIANA
G#RD - "}c. cu cls. I-VIII", Ocolna, MIHAELA MICU - "Lic. Teh. Al. Macedonski",
Meline]ti, Dolj, LIDIA EFTENIE - "}c. cu cls. I-VIII", Bor[scu, Gorj, DOINA STOICA ]i

26 Revista de matematic[ din Craiova


MIRCEA MARIO STOICA - Arad, AURICA ALDEA - "}c. cu cls. I-VIII Nr. 7", Foc]ani,
Vrancea, ELISABETA ALM{}AN - "}c. cu cls. I-VIII Nr. 3", Foc]ani, Vrancea, EMILIA
BATOG - "}c. cu cls. I-VIII Nr. 3", Foc]ani, Vrancea, MARIA COJANU - }c. cu cls. I-VIII,
Nr. 2 "Ion Basgan", Foc]ani, Vrancea, SILVIA CONDREA - }c. cu cls. I-VIII, Nr. 10"Duiliu
Zamfirescu", Foc]ani, Vrancea, PETRIA DOLIA - }c. cu cls. I-VIII, Nr. 10"Duiliu
Zamfirescu", Foc]ani, Vrancea, R{DI|A DR#MBU - }c. cu cls. I-VIII, Nr. 2 "Ion Basgan",
Foc]ani, Vrancea, LUCIA VIORICA GRIGORE - "}c. cu cls. I-VIII Nr. 7", Foc]ani,
Vrancea, MARIANA ICHIM - }c. cu cls. I-VIII, Nr. 2 "Ion Basgan", Foc]ani, Vrancea,
GEORGETA IFRIM - "}c. cu cls. I-VIII Nr. 3", Foc]ani, Vrancea, TITINA MICU - "}c. cu
cls. I-VIII Nr. 3", Foc]ani, Vrancea, ANETA R{DULESCU - Liceul Ped. "Spiru Haret",
Foc]ani, Vrancea, MIRELA RUSU - }c. cu cls. I-VIII, Nr. 5 "Anghel Saligny", Foc]ani,
Vrancea, TUDORI|A STEREA - "}c. cu cls. I-VIII Nr. 3", Foc]ani, Vrancea, LENU}A
STOICA - }c. cu cls. I-VIII, Nr. 2 "Ion Basgan", Foc]ani, Vrancea, FELICIA }TEFAN -
"}c. cu cls. I-VIII Nr. 7", Foc]ani, Vrancea, ZOI|A TABAN - }c. cu cls. I-VIII, Nr. 5
"Anghel Saligny", Foc]ani, Vrancea, GABRIELA T{B{CARU - }c. cu cls. I-VIII, Nr. 8
"Alexandru Vlahu\[", Foc]ani, Vrancea, NELA T{NASE - "}c. cu cls. I-VIII Nr. 3", Foc]ani,
Vrancea, MARI|ICA VI|{ - "}c. cu cls. I-VIII", Soveja, ELENA ZLOTA - }c. cu cls.
I-VIII, Nr. 9 "}tefan cel Mare", Foc]ani, Vrancea, FLORICA DONCA - }c. Gim."Grigore
Moisil", Satu-Mare. LUI|A R{DU| - }c. Nr. 37 "M. Eminescu", Craiova.

*
* *

Editura Cardinal 27
*********************************************************************
* probleme teme *
*********************************************************************

GIMNAZIU

Clasa a V-a
Probleme selectate de: prof. Cristiana Seinu, Craiova

T.G.4545. Calcula\i a  b ]tiind c[:


a  5  6  7  . . .  99, b  1  3  5  7  . . .  137.
T.G.4546. Fie S  15  21  27  33  . . .  213.
a) Afla\i c`\i termeni are S; b) Calcula\i S.
T.G.4547. a) Fie x  1  5  5 2  . . .  5 2013 . Ar[ta\i c[: x6.
b) Fie y  1  2  2 2  2 3  . . .  2 2009 . Ar[ta\i c[: 7  y.
T.G.4548. Demonstra\i c[ pentru orice n  N, n  1, avem:
a) 54  (2 n  3  2 n1  5  2 n2 );
b) 86  9  5  9 n ;
n2
c) 27  (2 n  3  2 n1  5  2 n2 ).
T.G.4549. a) Dac[ a  2b  16 ]i a  b  3c  53, calcula\i a  2c.
b) }tiind c[ a  2b  60 ]i ab  3d  2c , afla\i valoarea sumei S  3a 
11b  15d  10c  120.
T.G.4550. Calcula\i: x  (43 2011 : 1849 1005 ) : (2 100  2 99  2 98  . . .  2 2  2  1 ).
T.G.4551. Fie a  8  3 n2  25 n1 ]i b  7  5 n2  15 n1 , unde n  N.
i) Compara\i a cu b.
ii) Ar[ta\i c[ a ]i b dau acela]i rest la @mp[r\irea cu 165, pentru orice
num[r natural n.
T.G.4552. #ntre numerele 2 2016 ]i 5 369 se afl[ a numere naturale, iar @ntre numerele
3 144 ]i 11 246 se afl[ b numere naturale. Compara\i numerele a ]i b.
T.G.4553. Compara\i: i) a  3 4  44 3 ]i b  4 3  33 4 .
ii) a  33 44  444 333 ]i b  44 33  333 44 .
T.G.4554. Determina\i numerele prime a, b, c, d, astfel @nc`t 2 a  3b  2c  6d  56.

28 Revista de matematic[ din Craiova


T.G.4555. Afla\i numerele de forma: 2 m  3n cu m, n  N care au exact 15 divizori.
T.G.4556. Afla\i ultimele dou[ cifre ale num[rului b  3 n  3 n1  3 n2  . . .  3 n19 ,
unde n  N.
T.G.4557. Determina\i toate numerele de forma abc care @mp[r\ite la bc dau c`tul 4
]i restul bc  8.
T.G.4558. Afla\i toate numerele naturale n, care @mp[r\ite la 9 dau c`tul c ]i restul r,
iar @mp[r\ite la 5 dau c`tul r ]i restul c.
T.G.4459. Suma a dou[ numere naturale este 792. Dac[ ambele numere se @mpart la
4, se ob\in c`turi a c[ror diferen\[ este 122. Afla\i numerele.
T.G.4560. Afla\i num[rul natural a, ]tiind c[ restul @mp[r\irii lui a la 2013 este egal
cu restul @mp[r\irii lui 2013 la a.
T.G.4561. Un num[r natural n @mp[r\it la 9 d[ restul 7 ]i @mp[r\it la 5 d[ restul 2. Ce
rest ob\inem dac[ @mp[r\im num[rul n la 45?
T.G.4562. Se consider[ n numere naturale consecutive. Suma resturilor @mp[r\irii
celor n numere la 8 este 183. Afla\i num[rul n.
T.G.4563. a) Demonstra\i c[ num[rul abc este divizibil cu 8 dac[ ]i numai dac[
4a  bc este divizibil cu 8.
b) Determina\i numerele de forma 1a4a divizibile cu 8.
3
T.G.4564. Afla\i num[rul elementelor mul\imii A  abc 2  a, bc  7 1 .
4 5
n2
T.G.4565. Determina\i mul\imea: A  n  N 735 N .
7 3n  35 n
T.G.4566. Fie M o mul\ime de numere @ntregi, cu propriet[\ile:
a) 1  M.
b) x  M implic[ (x 2  x  1 )  M.
c) (x 2  3x  3 )  M implic[ x  M.
Demonstra\i c[: 13  M.
T.G.4567. Fie A  x  N 2 2005  x  2 2007 , B  y  N 3 2003  y  32005 .
Compara\i card A ]i card B.
T.G.4568. Fie mul\imile:
A  2 0 ; 2 0  2 1 ; 2 0  2 1  2 2 ; 2 0  2 1  2 2  2 3 ; . . . ,
B  3 0 ; 3 0  3 1 ; 3 0  3 1  3 2 ; 3 0  3 1  3 2  3 3 ; . . ..
Determina\i mul\imea A  B.
T.G.4569. Fie mul\imile: A  n 2  n  1  n  N ]i B  5k 2  1  k  N .
Ar[ta\i c[ A ]i B sunt disjuncte.

Editura Cardinal 29
Clasa a VI-a
Probleme selectate de: prof. Constantin Basarab, Craiova

T.G.4570. Determina\i mul\imile:


A  x  N 2x  3 48; B  x  N  (3x  1 )5 ]i x  50.
T.G.4571. Rezolva\i @n mul\imea numerelor naturale ecua\iile:
a) x 2 y  2x  35; b) xy  2x  3y  21.
T.G.4572. Ar[ta\i c[ num[rul a  2 30  3 15  4 14  3 15  12 13  4 este multiplu de 23.
T.G.4573. Demonstra\i c[ num[rul m  (n  18 )(2n  7 )(4n  1 ) este divizibil cu 3,
oricare ar fi n  N.
T.G.4574. Ar[ta\i c[ nu exist[ nici o pereche de numere naturale (m; n ), astfel @nc`t
num[rul a  4m  7  2n  3 s[ fie natural.
2m  4 3n  4
T.G.4575. Determina\i n  N, astfel @nc`t numerele: n  5, n  13, n  17, n  19 ]i
n  31 s[ fie simultan numere prime.
T.G.4576. Demonstra\i c[ num[rul n  3  3 2  3 3  . . .  3 2016  2015 este num[r
compus.
T.G.4577. Determina\i numerele naturale n, ]tiind c[ cel mai mic multiplu comun al
numerelor 3n  2 ]i 4n  3 este 165.
T.G.4578. Demonstra\i c[: 1  1  1  1  1  . . .  1  1  25 .
2 2 3 4 49 50 26
T.G.4579. Dac[ frac\ia 4n  3 este reductibil[ ]i n este num[r natural mai mic
6n  1
dec`t 50, afla\i valorile lui n.
T.G.4580. Calcula\i:
a) 1  1  1  60 ; b) 3  4  5  11 .
24 30 36 34 2  5 5  9 9  14 14  25
T.G.4481. Determina\i cifra a ]tiind c[:
a(a  1 )(a  2 )
a) 69  1, 2(a ); b) 350  7, (a5 ); c)  4, 68 .
54 46 50

T.G.4582. Determina\i frac\iile de forma ab , ]tiind c[: 3  ab  3 .


xy 14 xy 13

T.G.4583. Fie numerele:a  1  1 1  1 1  1 . . . 1  1 ]i


2 3 4 2016
b  1  1 1  1 1  1 ... 1  1 .
2 3 4 2016
Calcula\i a  b; b  a ]i a  b.

30 Revista de matematic[ din Craiova


7 8 9
T.G.4584. Calcula\i: a) 4  8 : 1637 ; b) 22  23  . . .  2016
2  1 .
9 27 3 3 3 3 27 672
T.G.4585. Fie punctele A, B, C, D, E situate pe dreapta d @n aceast[ ordine, astfel
@nc`t AC  6cm, BD  8cm, CD  4, 5cm ]i AE  16cm.
a) Se cer lungimile segmentelor: AB, BC, DE ]i BE.
b) Dac[ M este mijlocul lui BE, determina\i lungimile segmentelor:
AM, BM, CM, DM ]i ME.
T.G.4586. Fie punctele coliniare A, B ]i C @n aceast[ ordine, iar D ]i E, respectiv
simetricele lui A ]i B fa\[ de C.
Demonstra\i c[ AB  DE ]i AE  BD.
T.G.4587. Fie unghiul alungit AOB. De acea]i parte a dreptei AB se construiesc
semidreptele OC, OD ]i OE, astfel ca m AOC  12 m BOE , m BOE  23 m COE
]i m DOE  1 m BOE . Determina\i m[surile unghiurilor: AOC, COD, DOE, BOE,
2
AOD ]i COB.
T.G.4588. AOB ]i COD sunt unghiuri opuse la v`rf ]i OE este bisectoarea unghiu-
lui AOB. Dac[ m AOE  1 m BOC se cer m[surile unghiurilor: COD, DOE ]i AOD.
4
T.G.4589. Punctele A, O, B sunt situate pe dreapta d @n aceast[ ordine, iar punctele
M ]i N sunt situate de o parte ]i de alta a dreptei d, astfel @nc`t m BOM  42  32  ]i
m BON  137  28  .
a) Demonstra\i c[ MOB ]i AON sunt unghiuri opuse la v`rf.
b) Dac[ OP este bisectoarea unghiului AOM ]i OQ  OP, Q ]i P sunt de o
parte ]i de alta a dreptei d, se cer m[surile unghiurilor: QOP, QOM ]i PON.
T.G.4590. Pe laturile AB ]i AC ale triunghiului ABC se iau, respectiv punctele M ]i
N, astfel @nc`t AM  AN. Dac[ AMC  ANB ]i BN  CM  O, demonstra\i c[:
a) ABC este isoscel; b) OBC  OCB.
T.G.4591. Fie triunghiul ABC cu AB  AC, B  C, D  BC, E  BC, B este @ntre
D ]i C, iar C este @ntre B ]i E. Pe semidreapta AB se ia punctul F, iar pe AC se ia G,
astfel @nc`t BF  CG. Dac[ BD  CE, demonstra\i c[:
a) DF  EG ; b) AD  AE; c) FE  DG.
T.G.4592. Fie triunghiul ABC. Perpendiculara dus[ din B pe bisectoarea unghiului
A o intersecteaz[ @n P ]i pe AC @n D. Perpendiculara @n C pe AC se intersecteaz[ cu
AP @n E. Demonstra\i c[: a) AB  AD; b) BEA  DEA.
T.G.4593. Fie triunghiul ABC, E mijlocul lui BC, M  (AE ), BM  AC  P ]i
CM  AB  Q. Dac[ MB  MC, demonstra\i c[:
a) ABC este isoscel; b) MP  MQ.
T.G.4594. Fie triunghiul isoscel ABC de baz[ BC, M mijlocul lui AB, N mijlocul
lui AC ]i P simetricul lui M fa\[ de N. Demonstra\i c[:
a) CNP este isoscel; b) AP  BN.

Editura Cardinal 31
Clasa a VII-a
Probleme selectate de: prof. Dorina Dr[cea, Craiova

2012
T.G.4595. Calcula\i: 0, 125  2  1  0, (6 )  1  0, (4 ) .
3 8 64
1
T.G.4596. Dac[ A   32 ; 1 ; 0; 1; (6 ); 1024 ;  225 ;  2 1 ,
8 2 256 4
determina\i: A, A  N, A  Z, A  (Q  Z).
T.G.4597. Calcula\i:
3  3  3  ...  3  1  1  ...  1 .
2  5 5  8 8  11 2012  2015 12 23 2011  2012
3 3 32 7 2 108 72
T.G.4598. Calcula\i:  6 :   .
2 27 3 3 50 7

T.G.4599. Ar[ta\i c[: a) ( 2  3 )  (5  3 )  (1  2 )  N.


2 2 2

2 1 3  2 2025  2024
b)   ...   Q.
2 6 2025  2024
T.G.4600. Calcula\i:

a) a  a  a  a , a  0; b) 6  6  6  6  9 .

T.G.4601. Ar[ta\i c[: 1  1  2  3  . . .  2012  1007.


2 3 4 2013
T.G.4602. Rezolva\i ecua\iile: a) 2x  1  x  5  1  x  x  2 , x  Q.
3 2 4 3
b) (x  4)(x 2  8x  16)  0, x  Z; c) (x  2 )  x  y  10  0, x, y  Q.
2

T.G.4603. Determina\i patru numere ]tiind c[ suma lor este 360, primul este media
aritmetic[ dintre celelalte trei, al doilea este media aritmetic[ dintre al treilea ]i al
patrulea, iar al treilea este jum[tate din al patrulea.
T.G.4604. Ce sum[ a depus o persoan[ la banc[, ]tiind c[ banca acord[ o dob`nd[
de 12% anual, iar dup[ 3 luni persoana @ncaseaz[ o dob`nd[ de 36 de lei?
T.G.4605. Pre\ul unui obiect s-a redus cu 10%, iar dup[ o s[pt[m`n[ se mai reduce
cu @nc[ 10%. Dup[ cele dou[ reduceri obiectul cost[ 81 de lei.
Afla\i: a) Pre\ul ini\ial al obiectului.
b) Cu ce procent din pre\ul ini\ial s-a redus @n total pre\ul obiectului.
T.G.4606. Un turist parcurge o distan\[ @n trei etape: @n prima etap[ el parcurge 35%
din drum, iar @n a doua etap[ 20% din ce a mai r[mas, iar @n a treia etap[, restul de
624 km. Afla\i lungimea traseului.

32 Revista de matematic[ din Craiova


T.G.4607. Fie mul\imea: A  5  2 ; 4  2 ; 3  2 ; 2  2 ; 1  2 ; 1  2 ;
2  2 ; 3  2 ; 4  2 ; 5  2 .
a) Afla\i produsul elementelor lui A.
b) Afla\i suma inverselor elementelor din A.
T.G.4608. Fie ABCD un paralelogram,AC  BD  0, E mijlocul lui AB, AC  DE 
 F. Dac[ FO  2 cm, afla\i AC.
T.G.4609. Fie ABCD un paralelogram, E mijlocul lui AB, AC  BD  0, AB 
 20 cm, OE  10 cm ]i m A  60  .
Afla\i: a) P ABCD ; b) m[surile unghiurilor paralelogramului; c) BD.
T.G.4610. Fie triunghiul ABC echilateral cu P ABC  60 cm. Dac[ D ]i E sunt sime-
tricele lui B ]i C fa\[ de AC ]i respectiv AB, ar[ta\i c[:
a) E, A, D coliniare; b) EB  AC; c) afla\i ED.
T.G.4611. Fie ABCD un p[trat, triunghiul BCM echilateral, cu M un punct @n interi-
orul p[tratului ]i triunghiul DCN echilateral, cu N @n exteriorul p[tratului. Ar[ta\i c[:
a) MN  AC; b) AM  MN  AN.
T.G.4612. Dac[ ABCD este un trapez isoscel cu m ABC  60  , AB  20 cm, CD 
 12 cm, afla\i perimetrul trapezului.
T.G.4613. Fie ABCD un trapez dreptunghic cu m A  90  , AB  CD, AB  CD,
BC  AB  CD ]i E  BC, astfel @nc`t BE  BA. Perpendiculara @n E pe BC, inter-
secteaz[ pe AD @n F. Demonstra\i c[ FE  FA  FD.
T.G.4614. Fie ABCD un romb cu perimetrul egal cu 32 cm, iar M, N, P ]i Q mijloa-
cele laturilor AB, AD, DC ]i respectiv BC.
a) Demonstra\i c[ MNPQ este dreptunghi.
b) Dac[ m BAD  60  , afla\i QP.
T.G.4615. Fie ABCD dreptunghi, punctele M, N, P  (AB ) ]i punctele E, F, G  (DC),
astfel @nc`t AM  MN  NP  PB  CG  GF  FE  ED. Demonstra\i c[ dreptele NF, EP,
GM, AC ]i BD sunt concurente.
T.G.4616. Fie ABCD un p[trat ]i Q mijlocul lui BC. Dac[ AQ  CD  E ]i P
este mijlocul lui AD, ar[ta\i c[: A PQE  1 A ABCD .
4
T.G.4617. ABCD este p[trat, iar M ]i N sunt mijloacele laturilor AB, respectiv
BC. Paralela prin C la AN intersecteaz[ pe AB @n punctul E, iar AM  CN  I.
a) Demonstra\i c[ DE  AC. b) Demonstra\i c[ B, I, D coliniare.
T.G.4618. Fie ABCD trapez isoscel, AB  CD, m ABC  60  , M mijlocul AB.
Dac[ triunghiul MBC este echilateral, demonstra\i c[:
a) DM  AC; b) BD  AD; c) BD  MC.
T.G.4619. Fie ABCD un p[trat ]i E un punct @n interiorul s[u, astfel @nc`t  AEB
este isoscel.
a) Demonstra\i c[  DEC este isoscel.
b) Dac[ m AEB  120  , stabili\i natura  DEC.

Editura Cardinal 33
Clasa a VIII-a
Probleme selectate de: prof. Constantin Cazacu, Craiova
prof. Sidonia Cazacu, Craiova
T.G.4620. S[ se arate c[:
a) ( n  2  n )  n  1  (n  2 )n  2 , n  N.

b) ( 7  5 ) 6  35  2 .
2x  3 5 3a  2
T.G.4621. Rezolva\i ecua\ia:    7.
6 6 2 2 3 2 6

T.G.4622. Determina\i mul\imea A  n  N 15  n  N.


T.G.4623. Ar[ta\i c[:
a) n 2  n  1  R  Q; b) n 3  n  2  R  Q.
T.G.4624. a) Determina\i mul\imea: A  x  R 2  x  3  4.
b) G[si\i cel mai mare ]i cel mai mic element al mul\imii A.
T.G.4625. Dac[ x  (3; 4 ), ar[ta\i c[:
E  x  4  x  4  x  5  x  5  x  6  x  6 este num[r natural.
T.G.4626. Determina\i num[rul natural n, nenul, pentru care:
1  1  1  ...  1  6.
1 2 2  3 3  4 n1  n
T.G.4627. Rezolva\i ecua\iile: a) ax  4  a2  2x, a  R;
b) 3(x  2 )  3  2 2 (1  x ); c) (3x  4 )  (2x  1 )  5(x  5 )(x  5 ).
2 2

T.G.4628. Calcula\i media aritmetic[ ]i media geometric[ a numerelor:


1
1 1 5  7
a  : ;
5  7 5  7 7
7  5
1
b  2( 5  3 )  7  3  :  5.
5
T.G.4629. Determina\i mul\imile:
A  x  R x  2  2; B  x  R 3x  1  x  10.

T.G.4630. Dac[ E(x )  3x 2  18x  35 , determina\i valoarea maxim[ a lui E(x ),


2

x  6x  10
x  R.
T.G.4631. Se consider[ expresia:
: 4x 2  1  1 .
2
E(x )  1  3  2
2x  1 1  4x 2 2x  1 4x  1
a) Afla\i x  R, pentru care E(x ) este definit[;

34 Revista de matematic[ din Craiova


b) Ar[ta\i c[ E(x )   1 ;
4x
c) Afla\i cea mai mic[ valoare a lui x, pentru care E(x)  N.
T.G.4632. Afla\i cardinalul mul\imii A  x  Z  x  n  n  10 , n  Z .
2

2n  1
T.G.4633. Fie expresia:
E(x )  2 x  2  x
2
: 1  3 , x  R  2; 1.
x  4x  4 x  2 x  2 x2  4
a) Ar[ta\i c[ E(x )  4  2x ; b) Afla\i x  N, pentru care E(x )  0.
x2
c) Afla\i x  N, pentru care E(x )  2.

T.G.4634. Se d[ expresia E(x )  x 2 4x  10 .


2

x  4x  7
a) Ar[ta\i c[ E(x)  1; 2, oricare x  R;
b) Calcula\i partea @ntreag[ a lui E(x ).
c) Afla\i x  Z pentru care E(x)  Z.
T.G.4635. #n cubul ABCDA  B  C  D  , O este centrul fe\ei ABCD ]i M este centrul fe-
\ei ABB  A  .
a) Ar[ta\i c[ MO  (BCC  ).
b) Afla\i m[sura unghiului dintre MO ]i A  C  ]i m[sura unghiului dintre
 
D B ]i MO.
T.G.4636. Se consider[ paralelogramul ABCD ]i dreptele AA   BB   CC   DD  ,
astfel @nc`t punctele A  ]i B  sunt de aceea]i parte fa\[ de planul (ABC ), iar punctele
C  ]i D  sunt de cealalt[ parte fa\[ de planul (ABC ). Dac[ AA   6 cm, CC   18 cm,
BB   24 cm ]i DD   36 cm, stabili\i pozi\ia dreptelor A  C  ]i B  D  .
T.G.4637. Pe planul p[tratului ABCD se ridic[ perpendiculara MA, astfel @nc`t
MA  10 cm ]i A MBD  60 cm 2. Afla\i MC ]i d(A, (MBD )).
T.G.4638. Pe planul triunghiului dreptunghic ABC cu m(A )  90  , m(C )  15 
]i AB  2 cm. Se ridic[ perpendiculara AM, AM  4 cm. Calcula\i MC.
T.G.4639. #n tetraedrul regulat ABCD cu muchia de 6 cm, punctul M se afl[ pe
@n[l\imeaOA, O  (BCD ) astfel @nc`t MD  (MBC ). Afla\i d(M(ABC )).
T.G.4640. #n prisma triunghiular[ regulat[ ABCA  B  C  cu baza ABC punctele M ]i
N sunt mijloacele lui A  B  , respectiv A  C  . Afla\i sinusul unghiului format de A  B
cu planul (MNC ).
T.G.4641. #n prisma triunghiular[ regulat[ ABCA  B  C  , AB  6 cm, AA   18 cm ]i
punctul M este mijlocul lui CC  .
a) Afla\i tangenta unghiului dintre MB ]i planul (ABA  ).
b) Calcula\i tangenta unghiului dintre planele (AMB ) ]i (ACC  ).
T.G.4642. #n piramida patrulater[ regulat[ VABCD de baz[ ABCD, AB  8 cm ]i
VA  10 cm punctul M este mijlocul muchiei VB. Calcula\i:
a) Sinusul unghiului dintre MC ]i VD.
b) Sinusul unghiului dintre MC ]i planul (VAC ).

Editura Cardinal 35
T.G.4643. Fie d 1 ]i d 2 dou[ drepte paralele incluse @n planul  ]i A, B punctele fixe,
A, B  , AB    . Un plan oarecare , care con\ine dreapta AB intersecteaz[ d 1 @n
M 1 ]i d 2 @n M 2 . Ar[ta\i c[ dreapta M 1 M 2 trece printr-un punct fix.
T.G.4644. Prisma hexagonal[ regulat[ ABCDEFA  B  C  D  E  F  are AB  6 cm ]i
punctele M  EF  , N  EE  , P  DD  , Q  CC  , R  BB  , S  AA   , astfel @nc`t
FM  6 cm, EN  6 cm, DP  4 cm, CQ  2 cm, BR  2 cm ]i AS  4 cm. Ar[ta\i c[
punctele M, N, P, Q, R, S sunt puncte coplanare.

LICEU

Clasa a IX-a
Probleme selectate de: prof. Otilia Dr[gan , Craiova

T.L.3925. Ar[ta\i c[ x 2  10x  25  x 2  2x  1 este un num[r natural, pentru


orice x  (1, 5 ).
T.L.3926. Se consider[ numerele A  n  2 n  1 , B  n  2 n  1 , n  N,
n  2. Calcula\iA  B, B  A ]i B , unde a ]i a reprezint[ partea frac\ionar[,
A
respectiv @ntreag[ a num[rului a.
T.L.3927. Se consider[ num[rul x n  1  1  . . .  1 , n  N.
12 23 n (n  1 )
S[ se determine n, astfel @nc`t: x n   0, 99875.

T.L.3928. Rezolva\i @n R ecua\ia: x  1  x  2 .


2 3
T.L.3929. Determina\i mul\imile:
A  n  Z  3n  2  Z ]i B  n  Z  5n  1  Z .
n3 3n  2
T.L.3930. a) Demonstra\i prin induc\ie matematic[, c[ ( )n  2, ( )x 1 , x 2 , . . ., x n 
 R  : x 1  x 2  . . .  x n  x 1  x 2  . . .  x n , egalitatea av`nd loc dac[ ]i
numai dac[ cel pu\in (n  1 ) din numerele x 1 , x 2 , x 3 , . . ., x n sunt nule.
b) Rezolva\i @n R ecua\ia: 2x  1  3x 2  1  3x 2  2x .
T.L.3931. Demonstra\i c[:
1  1  1  ...  1  3  2n  1 , ( )n  2.
13 24 35 (n  1 )(n  1 ) 4 2n(n  1 )
T.L.3932. Studia\i monotonia ]i m[rginirea ]irului (a n ) n1 , n  N, a n  12  12 
1 2
 12  . . .  12 .
3 n

36 Revista de matematic[ din Craiova


T.L.3933. Stabili\i care dintre urm[toarele ]iruri (x n ) n1 sunt progresii aritmetice ]i
care sunt progresii geometrice, dac[:
a) x n1  x n  1 , x 1  2; b) x n1  1 x n , x 1  1;
2 2
x n 2
c) x n1 
3 , x 1  1; d) x n1  
2  xn, x1  2 .
T.L.3934. Calcula\i sumele:
a) 5  10  20  40  . . .  5120; b) 1  1  12  . . .  124 ;
2 2 2
1 1
c) 1    . . .  1 1 ; 1
d) 1    1 1 ...  1 .
2 4 8 1024 3 9 27 2187
T.L.3935. Rezolva\i @n R ecua\iile:
a) 3x  1  3x  4  3x  7  . . .  3x  37  121, (3 );
3 3 3 3
b) x  1  x  3  x  5  . . .  x  25  17, (3 ).
3x 3x 3x 3x
T.L.3936. Se consider[ ]irul (a n ) n1 , av`nd suma primilor n termeni S n  3n  7n .
2

2
a) Afla\i formula termenului general a n .
b) Ar[ta\i c[ (a n ) n1 este o progresie aritmetic[ ]i afla\i-i ra\ia.
c) Calcula\i: a k  a nk1 , k  N, k  n.
T.L.3937. Stabili\i natura patrulaterului ABCD dac[:

      
a) AD  BA  BC  DC;

      
b) AO  BO  DB  CB, unde O este intersec\ia diagonalelor.
T.L.3938. Se consider[ rombul ABCD de latur[ 6. M[sura unghiului ascu\it al rom-

  
bului este de 60  . Determina\i lungimea vectorului AB  AD.
T.L.3939. Consider[m paralelogramul ABCD ]i O centrul s[u. Fie M un punct din
 
 
plan, astfel @nc`t OM  AB ]i EF  BC, M  EF, E  AB, F  DC.
    
a) Ar[ta\i c[: MB  MC  MO.

   
 

b) Calcula\i AE  AD , ]tiind c[ AB  8, AD  2 3 ]i m(DAB )  30  .

T.L.3940. Dac[ triunghiurile ABC ]i A  B  C  au acela]i centru de greutate, ar[ta\i c[



    
AA  , BB  , CC  pot fi lungimile laturilor unui triunghi.

  
T.L.3941. Se consider[ triunghiul ABC ]i punctele M, N, P astfel @nc`t AM  MB,
 
     
BN  2NC ]i AP  2CP. Ar[ta\i c[ punctele M, N, P sunt coliniare.
T.L.3942. Pe laturile AB ]i AC ale triunghiului ABC se consider[ punctele M,

     
  
 
respectiv N, astfel @nc`t AM  3MB ]i AN  3 AC. Demonstra\i c[ vectorii MN ]i BC
4
sunt coliniari.

Editura Cardinal 37
       
T.L.3943. Se dau vectorii  a  i  2 j , b  2 i  j , c  (m  1 ) i  (2m  1 ) j ,
unde m  R.
 
a) Determina\i coordonatele vectorului 5 a  3 b .
 
b) Determina\i m  R, astfel @nc`t vectorii b , c s[ fie coliniari.

c) Determina\i m  R, astfel @nc`t c  10 .
 
T.L.3944. #n reperul O, i , j  se consider[ punctele A(2, 2 ), B(3, 5 ), C(4, 2 ).

Fie D  (BC ) astfel @nc`t semidreapta AD este bisectoarea BAC.


a) Determina\i coordonatele vectorului AD.
b) Determina\i coordonatele punctului D.
T.L.3945. Fie triunghiulABC ]i punctele M, N, P mijloacele laturilor BC, CA,
respectiv AB. Scrie\i sub o form[ mai simpl[:

  
  
   
  
a) AB  BC; b) AN  AP; c) MN  MP; d) MB  MN.
T.L.3946. Consider[m un triunghi ABC. Construi\i punctele M, N, P, astfel @nc`t:

    
    
 
AP  2PB  O ; BC  CM; NC  2AC.

    
Ar[ta\i c[: AM  BN  CP  O .
T.L.3947. Fie punctele A(2; 3 ); B(4; 5 ); C(1; 2 ). Calcula\i coordonatele vectorilor

 
    
AB, BC ]i 5AB  3AC.
T.L.3948. Fie punctele A(1; 0 ), B(0; 9 ) ]i C(3; 2 ).Afla\i coordonatele centrului de
greutate al triunghiului ABC.
T.L.3949. #n triunghiul ABC fie M mijlocul laturii AC ]i punctele M ]i P, astfel

       
@nc`t 8MN  BN  O ]i BP  4PC. Ar[ta\i c[ punctele A, N, P sunt coliniare.

Clasa a X-a
Probleme selectate de: prof. C[t[lin Spiridon, Craiova

T.L.3950. S[ se calculeze:

a) 13  30 2  9  4 2 ; b) 26  6 13  4 8  2 6  2 5 ;

c) 3 10  6 3  3 10  6 3 ; d) 3 5 2  7  3 5 2  7 .
n
T.L.3951. S[ se calculeze sumele: a) S   1 ;
k1 k  k1

c) S   lg k  1 .
n n
b) S   1 ;
k1 k k  1  (k  1 ) k k1 k

38 Revista de matematic[ din Craiova


2 1
x2  x x 3  4 y3 1
y2 6

T.L.3952. Se consider[ expresia E(x, y )   1   11 .


4 y  3 y 2 2 x 2
x3y5
S[ se calculeze E( 2 , 2 ).
T.L.3953. S[ se calculeze: a) log 2 2  6 log 4 6 2  2  5 log 2 5 2  2 ;
2
b) log 3 8  log 2 27  3 log 9 5
; c) 36 log 6 5
 10 1lg 2  3 log 9 36 .
T.L.3954. Fie x, y  (0,  ), x  3y, astfel @nc`t:
x  3y
lg  1 lg x 2  3 lg 6 x  2 lg 2.
2 4
x(7  13 )
S[ se calculeze: .
2y
T.L.3955. S[ se determine x, y  R, pentru care:
x  3  (y  3 )i
a) 1  1  1  i; b)  i.
xi yi (x  2 )  (y  4 )i
T.L.3956. S[ se calculeze modulele urm[toarelor numere complexe:
(1  i ) 2 (1  i ) 3 (2  i ) 6 (1  i 2 ) 2
a) z 1  ; b) z 2  .
(1  i 3 ) 4 (1  i 3 ) 2 (1  i 2 ) 8 (2  i ) 2
T.L.3957. S[ se determine z  C dac[:
a) z  3z  iz  8; b) 2z  z  z  4  2i;
c) z  3 z  1  19  4i; d) z  i  z  iz  z  1 ;
e) 2i z 2  3(1  i )z  3  i  0; f) z 2  (1  12i )z  (13  9i )  0.
T.L.3958. S[ se arate c[ pentru orice z 1 , z 2  C au loc rela\iile:
a) z 1  z 2  1 2  z 2  z 1 2  (1  z 1 2 )(1  z 2  z 2 );
b) 1  z 1  z 2 2  (1  z 1 2 )(1  z 2 2 ).
T.L.3959. Fie z k  C cu z k  1, k  1, n. S[ se arate c[:
(z 1  z 2 )(z 2  z 3 )  . . .  (z n1  z n )(z n  z 1 )
z1  z2  . . .  zn  R.
T.L.3960. S[ se reprezinte @n plan mul\imea punctelor de afix z care verific[:
a) z  1  2i  5; b) z  1  z  2  3i ; c) Im z  2  0.
z  4i
T.L.3961. a) Ar[ta\i c[ A(1  2i ), B(1  4i ) ]i C(2  7i ) sunt coliniare.
b) Ar[ta\i c[ A(1  4i ), B(2i ), C(3  2i ) ]i D(4 ) sunt v`rfurile unui para-
lelogram.
c) Ar[ta\i c[ A(4  2i ), B(2 ), C(1  3i ) ]i D(5  i ) sunt v`rfurile unui
dreptunghi.
T.L.3962. S[ se determine modulul ]i argumentul redus al numerelor:
(1  i ) 29 (1  i ) 15  (1  i 3 ) 20
a) z 1  ; b) z 2  .
( 3  i ) 17 (1  i 3 ) 31
T.L.3963. S[ se rezolve (@n C ) ecua\iile: a) (1  iz )  (1  iz )  0;
n n

Editura Cardinal 39
b) (z  1 )  (z  1 ) ; c) z 8  2z 4  2  0;
n n

d) z 10  (1  2i )  z 5  i  1  0; e) ( 3  i )  z 4  1  i  0.
T.L.3964. S[ se determine domeniul maxim de defini\ie al func\iilor:
1x b) f (x )  log x2 2 x  4
2
a) f (x )  ; ;
x  3x  2
2 x  5x  4
x1 1
c) f (x )  3 ; d) f (x )  arcsin(2x  x 2 ).
x2  9
T.L.3965. S[ se arate c[ urm[toarele func\ii sunt inversabile ]i s[ se determine
inversele lor: a) f : (0, )  (2, ), f (x )  3x  4 ;
b) f : 1,   3,  , f x )  x 2  x  1;
( ) ( ) ( c) f : R  R, f (x )  3 2x  1 ;
 3x  1, x  0
d) f : R 1  R 2, f (x )  2x  3 ; e) f : R  R, f (x )   2 .
x1  x  1, x  0
T.L.3966. Se consider[ func\ia f : R R, f (x )  x 3  3 x  1.
a) Ar[ta\i c[ f este injectiv[.
b) Rezolva\i @n R ecua\ia: x 3  3 x  514.
c) S[ se determine x  R,astfel @nc`t f (x )  3.
T.L.3967. Se consider[ func\ia f : R  R, f (x )  3 3x  2 .
a) Ar[ta\i c[ f este bijectiv[.
b) Rezolva\i @n R ecua\ia: 3  3 3x  2  x 3  2.
c) Rezolva\i inecua\ia f (x )  x.
T.L.3968. S[ se rezolve @n R ecua\iile:
a) 3x  7  x  1  2; b) 7  x  x  5  2;
b) x  7  2x  1  x ;
3 3 3 d) x 2  3x  x 2  3x  5  1;
e) 3 2  x  x  1  1; f) 3 x  2  x  1  3.
T.L.3969. S[ se rezolve @n R: a) 2  4 x  5  2 x  2  0;
b) 3 2x4  10  3 x1  1  0; c) 4 x x 3  5  2 x1 x 3  16  0;
2 2

d) 3  16 x  2  81 x  5  36 x ; e) 4 x1  7  10 x  3  25 x  0;
f) (2  3 )  (2  3 )  4; g) 2 x  3 x  4 x  9;
x x

h) 2 x  3 x  5 x ; i) 3 x2  1 .
x1
T.L.3970. S[ se rezolve @n R: a) log x1 (x 2  3x  14 )  2;
b) log 7 (x  2 )  log 7 (x  2 )  1  log 7 (2x  7 );
c) log 2 (x  1 )  log 2 (x  2 )  5;
d) log 2 (9 x1  7 )  2  log 2 (1  3 x1 );
e) log 2 x  log 2 (4x )  3 log 2 x  3;
2
f) 2 log 22 (4x 2 )  log 2 (8x )  1  0;
2
g) log 2 x  log sin 4 x  log 4 x 2  1;
h) 2  2 log 9 (2 x  2 )  log 3 2  1 log 3 (4 x1  1 );
2
 1;  1 .
2lg x lg 2 x4 lg x
(
i) lg x ) j) x
1000

40 Revista de matematic[ din Craiova


T.L.3971. S[ se rezolve @n R inecua\iile:
a) x 2  1  x; b) x  10  2  x;
c) 3 2x  4  3 x  3  0; d) log 13 x  log 13 (x  1 )  log 13 6;

e) log 2 (x  14 )  2 log 4 (x  2 )  2 log 12 1 ; f) x log 4 x1  256x.


8
T.L.3972. S[ se rezolve sistemele de ecua\ii:
  x  y 3
 x  y  2 
a)  ; b)  x y ;
 2x  y  3  4
  2 y  3 x  7

 2 x  3 y  5  x 2  2 y  8
c)  x1 ; d)  x  2 y1  10 ;
 2  3  13
y1

 log 3 (2 x  1 )  y  x lg y  10
e)  ; f)  .
 log 2 (3  5 )  x  1  (y  8 )  2
y lg x

T.L.3973. S[ se determine m  R, pentru care ecua\ia m  4 x  (2m  1 )  2 x  m  0,


are dou[ solu\ii distincte.
T.L.3974. S[ se determine m  R, pentru care ecua\ia 4 x  (m  1 )  2 x  m  0,
are exact o solu\ie real[.

Clasa a XI-a
Probleme selectate de: prof. C[t[lin Cristea, Craiova

1 2 3 4 5 6 7
T.L.3975. Fie permutarea    S7.
6 4 2 7 5 3 1
a) Determina\i num[rul de inversiuni ale permut[rii .
b) Demonstra\i c[ ecua\ia x 4   nu are solu\ii @n mul\imea S 7 .
c) Scrie\i permutarea  ca un produs de transpozi\ii.
2016
3 1
T.L.3976. Demonstra\i c[  2 2016  I 2 .
1 3
1 a1 a
T.L.3977. Fie matricea A  0 1 a  1 , a  R. Calcula\i An , n  N .
0 0 1
1 1 1 1
1 2 2 2
T.L.3978. Fie matricea A  .
1 2 3 3
1 2 3 4
a) Demonstra\i c[ matricea A este inversabil[.
b) Calcula\i inversa matricei A.

Editura Cardinal 41
3 1
T.L.3979. Rezolva\i @n M 2 (R ) ecua\ia: X 3  .
6 2
sin 10 sin 11 sin 12
T.L.3980. Calcula\i d  sin 20 sin 21 sin 22 .
sin 30 sin 31 sin 32
 x  y cos C  z cos B  0

T.L.3981. Demonstra\i c[ sistemul  x cos C  y  z cos A  0 , unde A, B, C sunt
 x cos B  y cos A  z  0

unghiurile unui triunghi oarecare, admite ]i solu\ii diferite de solu\ia nul[.
3 0 0
T.L.3982. Consider[m matricea A  0 2 0 .
0 0 1
a) Demonstra\i c[ matricea A este inversabil[.
b) Calcula\i inversa matricei A.
c) Determina\i num[rul solu\iilor ecua\iei X 2016  A, X  M 3 (C ).
1 2 2 1
T.L.3983. Fie matricele A  ]i B  , unde x  R.
3 x x 3
a) Demonstra\i c[ det(A  B )  det A  det B.
b) Pentru x  5 calcula\i B 1 .
c) Pentru x  6, demonstra\i c[ An  7n1  A, () n  N .
 x  y  az  1

T.L.3984. Se consider[ sistemul  x  ay  z  1, a  R ]i not[m cu A matricea
 ax  y  z  1

sistemului.
a) Demonstra\i c[ det A  (a  2 )  (a  1 ) .
2

b) Dac[ a  2, rezolva\i sistemul.


c) Dac[ a  2, demonstra\i c[ sistemul este incompatibil.
x2z x 1
T.L.3985. Se consider[ determinantul D(x, y, z )  y 2 z y 1 , x, y, z  R.
z z 1
( )
a) Calcula\i D 1, 2, 3 .
b) Demonstra\i c[ D(x.y.1 )  (x  y )(x  1 )(y  1 ).
c) G[si\i toate perechile (x, y )  Z  Z, x  y, astfel @nc`t D(x, y, 1 )  2016  (x  y ).
 ax  by  z  1

T.L.3986. S[ se discute ]i s[ se rezolve sistemul  x  ay  bz  1, a, b  R .
 ax  y  bz  1

an
T.L.3987. Fie ]irul (a n ) n1 , cu a 1  1 ]i a n1  .
an  2

42 Revista de matematic[ din Craiova


a) Demonstra\i c[ a n  n 1 , ( )n  N  .
2 1
b) Ar[ta\i c[ ]irul (a n ) n1 este convergent.
c) Calcula\i lim
n
(2 n a n ) n .
T.L.3988. Calcula\i:
a) lim
n
n3 n5  n5 n3 .

b) lim 1  1  ...  1 .
n
1 2 2 1 2 3 3 2 n n  1  (n  1 ) n

T.L.3989. Determina\i k  R ]tiind c[ lim


n
nk  n n  1  n n  1  R.
T.L.3990. Fie progresia aritmetic[ (a n ) n1 cu a 1 , r  0.
a) Demonstra\i c[ lim a  .
n n

b) Calcula\i lim 1  a1  a2
 ... 
an
.
n a n a1 a1  a2 a1  a2  . . . . an
T.L.3991. Calcula\i lim
n
( n  1   4n  3   9n  8 ), , ,   R.
T.L.3992. a) Determina\i parametrii reali a ]i b astfel @nc`t
lim
x
x 2  x  1  ax  b  0.
b) Determina\i parametrii reali m ]i n astfel @nc`t
lim
x
3
x 3  x  1  mx  n  2016.
2017x 2017x

lim
T.L.3993. Calcula\i: a) x x1 lim
, b) x 2x  1 .
x  x2  2 x  x2  2
x x
x1  x
T.L.3994. Determina\i a  R  1 astfel @nc`t lim
x
1  e2.
(a  1 ) 2 x 2  1

x 2  x  2  ax
T.L.3995. Determina\i a, b  R astfel @nc`t lim  2.
x1 xb 1
x 2  x  2  ax
T.L.3996. Determina\i a, b  R astfel @nc`t lim  b.
x1 x 2  3x  4
ln(x 2  px  1 )
T.L.3997. Determina\i p, q  R astfel @nc`t lim  2016.
x0 xq 1
T.L.3998. Fie f : R3  R, f (x )  x  ax  2017
2
, a  R.
(x  3 ) 2
a) Determina\i asimptotele la graficul func\iei f.
b) Determina\i valoarea lui a pentru care tangenta la graficul func\iei f @n
punctul de abscis[ 1 este paralel[ cu axa Ox.
T.L.3999. Ar[ta\i c[ dac[ a, b  R ]i 2 x  3sin x  ax  b  x3 , () x  (1, 1 ),
atunci a   ln 6.

Editura Cardinal 43
Clasa a XII-a
Probleme selectate de: prof. Raluca Ciurcea, Craiova

T.L.4000. Determina\i elementele simetrizabile @n raport cu opera\ia de @nmul\ire


din mul\imea Z14 .
T.L.4001. Afla\i elementul neutru al legii de compozi\ie “ ”, unde
x  y  xy  5x  5y  30, () x, y  R.
T.L.4002. Pentru legea de compozi\ie definit[ la T.L. 4001, calcula\i x  x  . . .  x.
de 2017 ori
T.L.4003. Pentru legea de compozi\ie definit[ la T.L. 4001, ar[ta\i c[ 1  2  3 
 . . .  2016  2017  5.
T.L.4004. Demonstra\i c[ legea de compozi\ie definit[ la T.L. 4001, induce pe Z o
structur[ de monoid comutativ ]i determina\i elementele simetrizabile ale acestuia.
T.L.4005. Determina\i o mul\ime de numere reale pe care opera\ia “ ” definit[ la
T.L. 4001, s[ induc[ o structur[ de grup abelian.
T.L.4006. Fie G  A  M3 (R)  det A  2k , k  Z.
a) Da\i dou[ exemple de elemente ale mul\imii G.
b) Demonstra\i c[ (G,  ) este grup.
 1 x 0 
 
T.L.4007. Fie mul\imea G   A x  ( ) 0 1 0 x  R .
 0 0 7x 
 
a) Da\i trei exemple de elemente ale mul\imii G.
b) Demonstra\i c[ opera\ia de @nmul\ire a matricelor induce o structur[ de
grup pe G.
c) Demonstra\i c[ grupurile (G,  ) ]i (R,  ) sunt izomorfe.
 1  5x 15x 
T.L.4008. Fie mul\imea G   A(x )  x  R   1 .
 x 1  3x 2 
a) Ar[ta\i c[ A(x )  A(y )  A(2xy  x  y ), oricare ar fi x, y  R  1 .
2
b) Demonstra\i c[ G @mpreun[ cu @nmul\irea matricelor p[tratice de ordinul
al II-lea formeaz[ o structur[ de grup abelian.
c) Demonstra\i c[ func\ia f : R   G, f (x )  A x  1 este izomorfism de la
2
(R  ,  ) la (G,  ).
T.L.4009. Fie (G,  ) grup.
a) Dac[ x 2 y 2  (xy ) , () x, y  G atunci (G,  ) este grup abelian.
2

b) Dac[ exist[ a  G, astfel @nc`t x 2  axa, ( ) x  G, atunci (G,  ) este grup


abelian.
1 2 3 4 5
T.L.4010. Afla\i ordinul elementului   @n grupul (S,  ) ]i calcula\i
3 5 4 1 2
 2017 ]i  1 .

44 Revista de matematic[ din Craiova


 a 2b 
T.L.4011. Fie mul\imea M   a, b  Z 3 .
 b a 
a) Determina\i cardinalul mul\imii M.
b) Demonstra\i c[ M  O 2  este subgrup al grupului multiplicativ al matri-
celor inversabile din M 2 (Z 3 ).
c) Afla\i elementele de ordinul al II-lea din M  O 2 .
T.L.4012. Afla\i ordinul elementului 5 @n grupul (Z7 ,  ).
T.L.4013. a) Verifica\i rela\ia: 1  1  1 , x  (1,  ).
2(x 2  1 ) 2(x 2  1 ) (x 2  1 )(x 2  1 )
b) Calcula\i:  4 1 dx, x  (1,  ).
x 1
T.L.4014. Determina\i func\ia f : R R, care admite o primitiv[ F : R R, F(x) 
 arctg 3 x 2  1  2017.
T.L.4015. Determina\i valorile parametrilor a, b  R pentru care func\ia
F : RR, F(x )  a x 2  3  b ln(x 2  3 ) este atiderivata func\iei f : R R, f (x ) 
x x2  3  1
 .
x2  3
T.L.4016. Afla\i func\ia F : RR al c[rei grafic taie axa Oy @n punctul de ordonat[
2(ln 2  1 ) ]i admite tangent[ @n orice punct de abscis[ x cu panta:
m 2 x3 .
(x  4 )( x 2  4  2 )

T.L.4017. a) Calcula\i  e x 3x3 dx;  e 3x  x 3 dx, x  (0,  ).


x 2 3x 2

e x e x
b) Calcula\i  1 2x 3x dx, x  (0,  ).
xx e
T.L.4018. Calcula\i urm[toarele integrale:
3
1 6 2

a)  2 1 dx; b)  1 dx; c)  1 dx.


0 x 3 1 x 3
2 0 3  x2
T.L.4019. Calcula\i:
2 5  2
a)  x  1 dx; b)  x 2  3x  2 dx; c)  cos x dx; d)  sin x dx.
0 0 0 0

T.L.4020. Calcula\i:
1 1 1
a)  ex
(1  e )(1  x 2 ) dx
x
; b)  x 2  5 dx; c)  ln x  x 2  5 dx.
1 0 0
1 1
T.L.4021. Consider[m ]irul (I n ) n N definit prin I 0  x  2017 dx ]i I n 
1 x n
dx.
0 0 x  2017
a) Calcula\i I 0  I 1 ;
b) Stabili\i monotonia ]irului (I n ) n N ;
c) Calcula\i lim I ]i lim
n n n
(nI n ).

Editura Cardinal 45
T.L.4022. Consider[m func\ia F : R  R, F(x )  arctg 1 .
x2  1
a) Determina\i func\ia f : R R a c[rei primitiv[ este F.
2
b) Calcula\i  f (x )dx.
0
T.L.4023. Calcula\i:

2
 (2 x  2x ) 2017 
5

a)  sin 2017 x  cos(2017x )dx; b) 3 x dx.


 5 2

T.L.4024. Stabili\i o rela\ie de recuren\[ pentru ]irul (I n ) n N  , unde:


1 2
a) I n  x n x 2  2 dx; b) I n  (2x  x 2 ) dx.
n

0 0

*
* *

46 Revista de matematic[ din Craiova


*********************************************************************
* LUCR{RI DE VERIFICARE *
*********************************************************************

CLASELE V-XII

Clasa a V-a
Lucrarea 1 - (timp de lucru: 50 min.) - Se acord[ 10 puncte din oficiu.
Alc[tuit[ de prof. CRISTIANA SEINU, Craiova

SUBIECTUL I. (45 puncte) - Pe foaia de tez[ se trec doar rezultatele.

1. Num[rul 3852 scris cu cifre romane este . . . . . .


2. G[si\i toate numerele naturale ab ]tiind c[ ab  2  ab  33. . . . . .
3. Rezultatul calculului 15  15 : 3 este . . . . . .
4. Compara\i numerele a  4 333 ]i b  2 444 mai mare este . . . . . .
5. Rezultatul calculului 2 2012 : 2 1006 : 2 503 scris ca putere este . . . . . .
6. Solu\ia ecua\iei 2  (x  1 )  4  x  10 este x  . . . . . .
7. Dac[ suma a dou[ numere este 35, iar unul dintre ele este 15, atunci cel de-al
doilea num[r este . . . . . .
8. Dac[ ab  15 ]i ac  10, atunci 2  a(b  c )  10  . . . . . .
9. Rezultatul calculului 17  2012  12  17  1000  17 este . . . . . .

SUBIECTUL II. (45 puncte) - Pe foaia de tez[ se trec doar rezultatele.

1. Calcula\i:
a) 1600 : 4 : 3  70  (2  45  3 2  2 3  5  3 : 12 ) : 5 3  2 2  95  2  12.
b) 3 2  3 3  2 54 : 2 52  (3 5 ) : 3 20  : 31.
4

2. Suma a trei numere este 1080. Primul num[r este de dou[ ori mai mare dec`t
al doilea ]i de trei ori mai mic dec`t al treilea. Afla\i numerele.
3. Ar[ta\i c[ num[rul A  2  2 1  2 2  2 3  2 4  2 5 este p[trat perfect.
4. Determina\i num[rul natural x ]tiind c[ este solu\ie a ecua\iei
(324  205 : 5  a ) : 6  3  127  307.

Editura Cardinal 47
Lucrarea 2 - (timp de lucru: 50 min.) - Se acord[ 10 puncte din oficiu.
Alc[tuit[ de prof. CRISTIANA SEINU, Craiova

SUBIECTUL I. (45 puncte) - Pe foaia de tez[ se trec doar rezultatele.


5p 1. Rezultatul calculului 15  2  6 este egal cu . . . . . .
5p 2. Dintre numerele 2011 ]i 2101 mai mic este num[rul . . . . . .
5p 3. Succesorul lui 205 este . . . . . .
5p 4. Produsul numerelor 208 ]i 67 este egal cu . . . . . .
5p 5. Num[rul de 15 ori mai mic dec`t 345 este egal cu . . . . . .
5p 6. Rezultatul calculului 2 3  5  2 0  1 5 este egal cu . . . . . .
5p 7. Restul @mp[r\irii num[rului 243 la 12 este egal cu . . . . . .
5p 8. Divizorii num[rului 16 sunt . . . . . .
5p 9. Solu\ia ecua\iei x : 6  7 este egal[ cu . . . . . .

SUBIECTUL II. (45 puncte) - Pe foaia de tez[ se trec doar rezultatele.

15p 1. Calcula\i: a) 102  61  50  6  194  3  (17  5  3 ).


b) 2 5  2  (2 3 ) : (2 5 )  2 0  1 9  9 1  0 9 .
4 4

10p 2. Media aritmetic[ a trei numere este 161. Afla\i numerele dac[ media arit-
metic[ a primelor dou[ este 148, iar al treilea este mai mare dec`t al doilea cu 28.
10p 3. Dac[ card (A )  24, card (B )  15 ]i card (A  B )  31, afla\i card (A  B ),
card (A  B ) ]i card (B  A ).
10p 4. Fie x, y, z numere naturale astfel @nc`t x  y  z  11.
a) Ar[ta\i c[ suma xy  yz  zx este p[trat perfect.
b) Ar[ta\i c[ nu exist[ x, y ]i z consecutive care s[ @ndeplineasc[ aceast[
condi\ie.

Clasa a VI-a
Lucrarea 1 - (timp de lucru: 50 min.)
Alc[tuit[ de prof. CONSTANTIN BASARAB, Craiova

SUBIECTUL I. (30 puncte) - Pe foaia de tez[ se trec doar rezultatele.


5p 1. x  N (2x  4)  60  . . . . . .
5p 2. Cel mai mare divizor prim al lui 2016 este . . . . . .
5p 3. Dac[ 2x  3  0, 6875, atunci x  . . . . . .
3x  4

48 Revista de matematic[ din Craiova


5p 4. Dac[ XOY ]i YOT sunt suplementare, ]i 2m XOY  3m YOT , atunci
m XOY  . . . . . .
5p 5. Dac[ AB  18 cm, C  AB, D  AB, astfel @nc`t C este mijlocul lui AD ]i D
este mijlocul lui CB, atunci lungimea lui CD este . . . . . . cm.
5p 6. Dac[ m[sura unui unghi este 48  10  30  , atunci m[sura complementului s[u
este . . . . . .

SUBIECTUL II. (30 puncte) - Pe foaia de tez[ se trec rezolv[rile complete.

10p 1. Rezolva\i ecua\ia: 31x  10 1, 75 : 5  1, (2 )  3 .


8 5
10p 2. Determina\i numerele naturale a ]i b, ]tiind c[ (a; b )  12 ]i 2a  3b  156.
10p 3. Determina\i numerele naturale n ]i p, ]tiind c[ p este num[r prim ]i
n 2  n  p  242.

SUBIECTUL III. (30 puncte) - Pe foaia de tez[ se trec rezolv[rile complete.

15p 1. Se dau punctele coliniare A, B, C, astfel @nc`t AB  12 cm, AC  40 mm ]i


BC  1, 6 dm. Dac[ M este mijlocul lui AC, N  (AB ) ]i MN  6 cm, determina\i
lungimea lui NB.
15p 2. AOB este unghi alungit, iar C ]i D sunt de o parte ]i de alta a dreptei AB,
astfel @nc`t m AOC  40  ]i OD  OC.
a) Afla\i m[surile unghiurilor BOC ]i BOD;
5p b) Afla\i m[sura unghiului dintre bisectoarele unghiurilor AOD ]i BOC.

Lucrarea 2 - (timp de lucru: 50 min.)


Alc[tuit[ de prof. MARLENA BASARAB, Craiova

SUBIECTUL I. (30 puncte) - Pe foaia de tez[ se trec doar rezultatele.

5p 1. Cardinalul mul\imii n  N  12  N este egal cu . . . . . .


n3
5p 2. Descompunerea @n factori primi a num[rului 32760 este . . . . . .
5p 3. Cel mai mare divizor comun al numerelor 256 ]i 364 este . . . . . .
5p 4. M[sura unghiului format de laturile necomune a dou[ unghiuri adiacente este
 
82 30 . M[sura unghiului format de bisectoarele lor este . . . . . .
5p 5. M[sura suplementului unghiului de 72  15  35  este . . . . . .
5p 6. Punctele A, B, C sunt coliniare @n aceast[ ordine, iar P ]i Q sunt mijloacele
segmentelor AB, respectiv BC. Dac[ AB  60 cm ]i AC  90 cm, atunci lungimea
segmentului PQ este . . . . . .

Editura Cardinal 49
SUBIECTUL II. (30 puncte) - Pe foaia de tez[ se trec rezolv[rile complete.
10p 1. Afla\i cel mai mic num[r natural care @mp[r\it pe r`nd la 24, 36 ]i 60 d[ res-
turile 19, 31, respectiv 55.
10p 2. Afla\i numerele prime a, b, c, astfel @nc`t a  30b  12c  86.
10p 3. Dac[ a, b  N ]i a  7b  17 , afla\i b.
b3

SUBIECTUL III. (30 puncte) - Pe foaia de tez[ se trec rezolv[rile complete.


15p 1. Pe dreapta d se iau punctele A, B, C, D, astfel @nc`t AD  20 cm, C  AD,
AC  4 cm ]i E mijlocul lui CD. Pe semidreapta opus[ semidreptei AD se ia punctul
F, astfel @nc`t AF  AE. Determina\i lungimile segmentelor CD, AE, AF, EF
]i FD.
15p 2. Fie unghiul ascu\it AOB, OE semidreapta opus[ semidreptei OA, OC  OA,
OD  OB, astfel @nc`t punctele B, C ]i D s[ fie de acea]i parte a dreptei OA.
Dac[ m DOE  2 m AOB ]i OF este bisectoarea unghiului AOD, calcula\i
m DOE ]i EOF .

Clasa a VII-a
Lucrarea 1 - (timp de lucru: 2 ore)
Alc[tuit[ de prof. DORINA DR{CEA, Craiova

SUBIECTUL I. (30 puncte) - Pe foaia de tez[ se trec doar rezultatele.

5p 1. 1, (3 )  1  (2 )  . . . . . .
2
5p 2.  13  . . . . . .
4
5p 3. Solu\ia ra\ional[ a ecua\iei: 2x  1  x  1 este . . . . . .
3 2
5p 4. Dac[ ABCD este paralelogram, AB  10 cm, P ABCD  30 cm, BC  . . . . . .
5p 5. Dac[ ABCD este romb, BD  8 cm, m A  60  , atunciP ABCD  . . . . . .
5p 6. Dac[ ABCD este p[trat, atunci m ABD  . . . . . .

SUBIECTUL II. (30 puncte) - Pe foaia de tez[ se trec rezolv[rile complete.

10p 1. Tat[l ]i fiul au @mpreun[ 56 de ani. Tat[l este de 2, 5 ori mai mare dec`t fiul.
a) Afla\i v`rsta tat[lui ]i a fiului.
b) Peste c`\i ani tat[l va avea v`rsta egal[ cu dublul v`rstei fiului?

50 Revista de matematic[ din Craiova


1
10p 2. Calcula\i: a) 3  0, (3 )  6   2 2 : 2  1 0.
5 8 3 15 2
b) (2 18  3 128  5 72 )  (4 2 ).
10p 3. Rezolva\i ecua\iile:
a) (2x  1 )  16, x  Q; b) 1  3(2x  0, 8)  0, 5  4x, x  Q.
2

SUBIECTUL III. (30 puncte) - Pe foaia de tez[ se trec rezolv[rile complete.


10p 1. Fie ABCD un trapez isoscel cu AB  DC, AD  BC  M, AB  20 cm.
a) Afla\i P MAB .
b) Dac[ CD  10 cm, afla\i P ABCD ]i demonstra\i c[ AC  BC.
5p 2. ABCD este paralelogram cu m A  60  ]i perimetrul egal cu 60 cm.
Dac[ BD  AD, afla\i lungimile laturilor paralelogramului.
15p 3. Fie ABCD un dreptunghi, M mijlocul lui AB, N mijlocul lui CD ]i AC 
 BD  O.
a) Demonstra\i c[ AMCN paralelogram.
b) Demonstra\i c[ AC, BD ]i MN concurente.
c) Demonstra\i c[ AN  DM  BN  CM.

Lucrarea 2 - (timp de lucru: 50 min.)


Alc[tuit[ de prof. DORINA DR{CEA, Craiova

53 49 1
1. Calcula\i: a)  1 : 1  3  2  1  0, (3 ) 5 : 0, (3 ) 4  2 .
2 2 2
b) x  y, unde x  12  8  24 ]i y  18  54  27 .
2. Rezolva\i ecua\iile: a) 1 (x  1 )  10   1 x  1  2, x  Q.
2 3 2
b) x  2  3x  2  0, x  Q.
3. Trei copii au @mpreun[ 1560 lei. Dup[ ce au cheltuit: primul 75% din suma
sa, al doilea 80%, iar al treilea 2 din suma sa, au mai r[mas sume egale.
3
a) Ce sum[ a avut fiecare copil?
b) Ce sum[ a cheltuit fiecare copil?
4. ABCD este trapez isoscel, AB  CD, AB  10 cm, CD  6 cm ]i AC  BD.
Afla\i @n[l\imea trapezului ]i aria acestuia.
5. ABCD este paralelogram cu AB  6 cm ]i BC  2 cm.
a) Demonstra\i c[ bisectoarele unghiurilor DAB ]i ABC @mpart latura CD
@n trei segmente congruente.
b) Determina\i distan\a de la M la mijlocul lui AB, unde M este intersec\ia
celor dou[ bisectoare.

Editura Cardinal 51
Clasa a VIII-a
Lucrarea 1 - (timp de lucru: 120 min.)
Alc[tuit[ de prof. CONSTANTIN CAZACU, Craiova

SUBIECTUL I. (30 puncte) - Pe foaia de tez[ se trec numai rezultatele.

5p 1. Rezultatul calculului 27 : ( 3 ) este egal cu . . . . . .


5p 2. Dintre numerele 7 3 ]i 13 mai mic este num[rul . . . . . .
5p 3. #n intervalul 2; 5 se afl[ un num[r de . . . . . . numere naturale.
5p 4. Rezultatul calculului 24x 2 : (2x  x  x ) este egal cu . . . . . .
5. Prisma patrulater[ regulat[ dreapt[ ABCDA  B  C  D  are fe\ele laterale p[trate
cu latura de 3 cm.
5p a) Suma lungimilor tuturor muchiilor prismei este . . . . . . cm.
5p b) M[sura unghiului dintre dreptele BC  ]i AD este egal[ cu . . . . . . grade.

SUBIECTUL II. (30 puncte) - Pe foaia de tez[ se trec rezolv[rile complete.

5p 1. Desena\i o piramid[ patrulater[ regulat[ de v`rf V ]i baza ABCD.


2. Fie numerele reale: a  (2  3 ) ]i b  (2  3 ) .
2 2

5p a) Calcula\i media aritmetic[ a numerelor a ]i b.


5p b) Calcula\i media geometric[ a numerelor a ]i b.
5p 3. Ar[ta\i c[ num[rul 3  12  4 este num[r @ntreg.
12  3
5p 4. Dac[ x  1x  6, afla\i x 4  14 .
x
5p 5. Ar[ta\i c[ num[rul a  17  12 2  6  4 2  2 este num[r natural.

SUBIECTUL III. (30 puncte) - Pe foaia de tez[ se trec rezolv[rile complete.

1. O cutie are forma unui paralelipiped dreptunghic ABCDA  B  C  D  cu AB 


 4 3 dm, BC  4 dm ]i AA   8 dm. O furnic[ se afl[ @n punctul A, iar @n punctul C 
se afl[ o buc[\ic[ de zah[r la care vrea s[ ajung[ furnica.
5p a) Afla\i lungimea drumului minimpe care @l poate alege furnica, dac[ ea va
merge numai pe muchiile paralelipipedului.
5p b) Afla\i lungimea drumului minim pe care @l poate alege furnica, dac[ ea va
merge pe suprafa\a paralelipipedului.
5p c) Afla\i distan\a de la punctul A la planul (A  DB ).

52 Revista de matematic[ din Craiova


2. Se consider[ piramida patrulater[ regulat[ VABCD cu v`rful V ]i AB  8 cm,
AV  4 3 cm. Dac[ punctul M este mijlocul lui VD ]i punctul N este mijlocul lui
BC.
5p a) Afla\i P MNC .
5p b) Calcula\i distan\a de la A la planul (VBC ).
5p c) Afla\i distan\a de la punctul M la dreapta BC.

Lucrarea 2- (timp de lucru: 120 min)


Alc[tuit[ de prof. SIDONIA CAZACU, Craiova

SUBIECTUL I. (30 puncte) - Pe foaia de tez[ se trec numai rezultatele.


5p 1. Rezultatul calculului 2 2  8 este . . . . . .
5p 2. Media geometric[ a numerelor 2 ]i 8 este . . . . . .
5p 3. Partea @ntreag[ a num[rului 5, 2 este . . . . . .
5p 4. Simplific`nd frac\ia 2x2  2 se ob\ine . . . . . .
x 1
5p 5. #n cubul ABCDA  B  C  D  m[sura unghiului dintre B  C ]i planul (ABC ) are
m[sura de . . . . . . grade.
5p 6. Num[rul muchiilor unui tetraedru regulat este . . . . . .

SUBIECTUL II. (30 puncte) - Pe foaia de tez[ se trec rezolv[rile complete.

5p 1. Desena\i o piramid[ hexagonal[ regulat[ VABCDEF cu v`rful V.


2. Ar[ta\i c[ (2x  1 )  (x  1 )  (x  2 )(x  2 )  3x 2  13  (x  3 ) .
2 2 2
5p
1
4 5
5p 3. Calcula\i:  3 : 3 .
5 2 5 20
5p 4. Fie A  x  R 2x  1  5. Determina\i A  Z.
5. Fie expresia E(x )  x2 1 : x  3  1 1
10p
x  1 4x  4 1  x  1  x  1 , x  R 1, 1.
a) Ar[ta\i c[ E(x )  24x ;
x 1
b) Determina\i valorile @ntregi ale lui x pentru care E(x )  Z.

SUBIECTUL III. (30 puncte) - Pe foaia de tez[ se trec rezolv[rile complete.

1. Se consider[ paralelipipedul dreptunghic ABCDA B C D cu AB  4 2 cm,


BC  4 3 cm ]i AA  4 cm. Calcula\i:
5p a) A  C;
5p b) m((A  C, (AA  B )));
5p c) d(A  , BD ).

Editura Cardinal 53
2. Se consider[ piramida triunghiular[ regulat[ VABC cu baza ABC ]i M, N, P
sunt mijloacele muchiilor AB, BC, respectiv VB. Dac[ VA  4 cm, AB  4 2 cm,
5p a) ar[ta\i c[ (MNP )  (VAC );
5p b) afla\i m((VA, MN ));
5p c) afla\i m((VM, AC )).

Clasa a IX-a
Lucrarea 1 - (timp de lucru: 90 min.)
Alc[tuit[ de prof. RALUCA CIURCEA, Craiova

SUBIECTUL I. (45 puncte)


1. Rezolva\i @n R ecua\iile:
7,5p a) x  1  x  3  2;
7,5p b) x  1  4  2x ;
3 5
7,5p c) (x 3  x )(x  2 )  24.
2. Demonstra\i c[:
7,5p a) x 4  x 2  2x  2  0, () x  R;
n(n  1 )(n  2 ) ( )
7,5p b) 1  2  2  3  . . .  n(n  1 )  ,  n  N.
3
7,5p c) 3 n  4n  1, () n  N, n  2.

SUBIECTUL II. (45 puncte)

1. Fie punctele A(3, 1 ); B(0, 4 ); C(4, 1 ).


7,5p a) Calcula\i coordonatele ]i modulele vectorilor:

       
AB, AC, 3BC  7AC  2AB.
7,5p b) Calcula\i coordonatele centrului de greutate al triunghiului ABC.
 
  
7,5p c) Dac[ BP  2PC  O , ar[ta\i c[: 3AP  AB  2AC.
2. Fie un triunghi ABC, AB bisectoarea unghiului ABC, D  (AC ), M mijlocul

 

segmentului (BC ) ]i N un punct, astfel @nc`t DN  2 NB, AB  4 cm, BC  6 cm, AC 
5
 7 cm.

 
  
7,5p a) Exprima\i vectorul BD @n func\ie de vectorii AB ]i BC.

 
  
7,5p b) Exprima\i vectorul AN @n func\ie de vectorii AB ]i AD.
7,5p c) Demonstra\i c[ punctele A, N, M sunt coliniare.

54 Revista de matematic[ din Craiova


Lucrarea 2 - (timp de lucru: 90 min.)
Alc[tuit[ de prof. DANIEL-CRISTIAN CIURCEA, Craiova

SUBIECTUL I. (45 puncte)

1. Rezolva\i @n R ecua\iile:
7,5p a) 2x  3  7  x;
7,5p b) x  4x  5;
3
7,5p c) (x  2 )(2  x )  3x.
2.
7,5p a) Rezolva\i @n R inecua\ia 6  3x  7.
7,5p b) Demonstra\i c[ a 3  b 3  ab(a  b ), ( ) a, b  (0,   ).
n(n  1 )(n  5 ) ( )
7,5p c) Demonstra\i c[ 1  4  2  5  . . .  n(n  3 )  ,  n  N.
3
SUBIECTUL II. (45 puncte)

1. Fie punctele A(5; 0 ), B(1; 7 ) ]i C(3, 1 ).


7,5p a) Calcula\i coordonatele ]i modulele vectorilor:

 
      
2AB, BC, 3BC  5AC  BA.
7,5p b) Calcula\i coordonatele mijloacelor laturilor triunghiului ABC.
7,5p c) Dac[ G este centrul de greutate al triunghiului ABC, exprima\i vectorul

 
  
GA @n func\ie de vectorii AB ]i AC.

   
  
2. Fie un triunghi ABC ]i punctele M, N, P, astfel @nc`t 3AC  4MC; NA  2NC 
    
 0 ]i AP  6PM.

 
  
7,5p a) Exprima\i vectorul BP @n func\ie de vectorii BA ]i BM.
7,5p b) Ar[ta\i c[ 3BN  2BC  BA.
7,5p c) Demonstra\i c[ punctele B, P, N sunt coliniare.

Editura Cardinal 55
Clasa a X-a
Lucrarea 1 - (timp de lucru: 90 min.)
Alc[tuit[ de prof. C{T{LIN SPIRIDON, Craiova

SUBIECTUL I.

1. S[ se arate c[: 6  2 5  6  2 5  Q.
2. S[ se ordoneze cresc[tor numerele: log 2 3, 2 ]i 3 3 .
3. S[ se rezolve @n R ecua\ia: 3 x1  3 1x  10.
4. S[ se rezolve @n R ecua\ia: log 2 (4x )  log 2 x  3.
2
5. S[ se determine z  C, dac[ z  z  z  i.
(1  i ) 2016
6. Ar[ta\i c[:  N.
(1  i 3 ) 1002

SUBIECTUL II.

Se consider[ func\ia f : D  R, D  R, f (x )  log x1 2 2x  2 .


x  5x  6
a) S[ se determine domeniul maxim de defini\ie D.
b) S[ se rezolve ecua\ia f (x )  1.
c) S[ se rezolve inecua\ia f (x )  0.

SUBIECTUL III.

a) S[ se rezolve @n C ecua\ia: (1  i )z 2  (i  6 )z  2  3i  0.
b) S[ se determine z  C, astfel @nc`t: (1  i 3 )z 4  1  i  0.
c) S[ se calculeze: cos   cos 2  . . .  cos 6 .
7 7 7

Lucrarea 2 - (timp de lucru: 90 min.)


Alc[tuit[ de prof. CRISTINA SPIRIDON, Craiova

SUBIECTUL I.

1. S[ se calculeze 3 1log 3 10  4 1log 4 5  6 log 6 91 .


2. S[ se ordoneze cresc[tor: 3 , 3 9 , 4 17 .
3. S[ se rezolve @n R ecua\ia: 2 x  4  8  2 x  2.
4. S[ se rezolve @n R ecua\ia: 1  1  1.
log x 6 log x1 6

56 Revista de matematic[ din Craiova


5. S[ se determine z  C, astfel @nc`t: z  z  z  10  12i.
2001 2001
1 3 1 3
6. Ar[ta\i c[:   i   i  2.
2 2 2 2

SUBIECTUL II.

Se consider[ func\ia f : R  R, f (x )  3 4x  3 .
a) Ar[ta\i c[ f este inversabil[ ]i afla\i inversa ei.
b) Rezolva\i @n R ecua\ia: 4  3 4x  3  x 3  3.
c) Rezolva\i @n R ecua\ia: 2 f (x)  log 2 f (x )  2.

SUBIECTUL III.

a) S[ se rezolve @n C ecua\ia: 1  ix  1  2i .
4

1  ix 1  2i
b) S[ se determine z  C, astfel @nc`t: z  10z 2  9  0.
4

c) S[ se calculeze: cos   cos 2  . . .  cos 8 .


9 9 9

Clasa a XI-a
Lucrarea 1 - (timp de lucru: 90 min.)
Alc[tuit[ de prof. C{T{LIN CRISTEA, Craiova

SUBIECTUL I.
a1 a2 1 1
b1 b2 1 1
1. a) Calcula\i determinantul: d  .
c1 c2 1 1
d1 d2 1 1
b) Consider[m urm[torii determinan\i cu elemente numere reale:
a1 a2 1 b1 b2 1 c1 c2 1 d1 d2 1
d1  b1 b2 1 ; d1  c1 c2 1 ; d1  d1 d2 1 ; d1  a1 a2 1 .
c1 c2 1 d1 d2 1 a1 a2 1 b1 b2 1
Se ]tie c[ trei dintre ei sunt egali cu 1. Demonstra\i c[ si cel de-al patrulea este egal
cu 1.

Editura Cardinal 57
 x  my  z  1

2. Se consider[ sistemul:  mx  y  z  1, m  R .
 x  y  mz  1

a) Ar[ta\i c[ determinantul matricei sistemului este egal cu m 3  3m  2;
b) S[ se determine m  R pentru care sistemul este incompatibil;
c) S[ se determine m  R ]tiind c[ sistemul admite o infinitate de solu\ii.

SUBIECTUL II.

1. S[ se calculeze:
a) lim 1  cos
2
x;
2
x0 sin x
b) lim
x
9x 2  x  3  3x .
 3 x  1  2be x , x  0

2. Fie func\ia f : (0,  ) f : R  R, f (x )   ax  b, x0 .
 (2 x  a ) ln(x  e ), x  0

a) Determina\i a, b  R astfel @nc`t f s[ fie continu[ pe R.
f (x )
b) Calcula\i lim
x x  3 x
.
f (x )(x  2 )
c) Pentru a  0, rezolva\i pe intervalul (0,  ) inecua\ia 2  0.
2  512

Lucrarea 2 - (timp de lucru: 120 min.)


Alc[tuit[ de prof. C{T{LIN CRISTEA, Craiova

SUBIECTUL I. (30 puncte)

0 1 m
1. Se consider[ matricea A  1 2 m , m  R.
1 2 1
5p a) Determina\i valorile parametrului real m pentru care matricea A este in-
versabil[.
5p b) Pentru m  2 determina\i inversa matricei A.
5p c) Ar[ta\i c[ dac[ m  0 atunci A k  O 3 , pentru orice k  N .
2. Se consider[ sistemul de ecua\ii liniare:
 x  y  (3  2a )z  1
 (
 2a  3 )x  y  (2a  3 )z  4  2a, a  R .
 (2a  3 )x  3y  3z  1


58 Revista de matematic[ din Craiova


5p a) Calcula\i determinantul matricei sistemul.
5p b) Determina\i valorile lui a  R pentru care matricea asociat[ sistemului
are rangul minim.
5p c) Dac[ a  1, rezolva\i sistemul.

SUBIECTUL II. (30 puncte)

 x 3  ax  b, dac[ x  1
 x1
1. Se consider[ func\ia f : R  R, f (x )   ae , dac[ x  1 .
 x 2  2x  2 , dac[ x  1

5p a) S[ se determine parametrii reali a ]i b astfel @nc`t f s[ fie continu[ @n
x  1.
f (x )
5p b) Pentru a  3 ]i b  2 rezolva\i inecua\ia  0.
2x  1
x 2
f (x )
5p c) Calcula\i lim
x x .
1
]i (b n ) n 1 definite prin a n lim
2. Fie ]irurile (a n ) n1 x
(1  x sin nx ) x3 , b n  a 1 
a 2  . . .  a n .
5p a) Calcula\i a n .
5p b) Demonstra\i c[ ]irul (b n ) n1 este convergent.
5p c) Calcula\i lim
n n
b .

Clasa a XII-a
Lucrarea 1 - (timp de lucru: 2 ore) - Se acord[ 10 puncte din oficiu.
Alc[tuit[ de prof. RALUCA CIURCEA, Craiova

SUBIECTUL I (45 puncte)

1. Pe R definim opera\ia x  y  xy  2x  2y  2, oricare ar fi x, y  R.


5p a) Calcula\i x  y  (x  2 )  (y  2), x, y  R.
10p b) Demonstra\i c[ G  (2,  ) este parte stabil[ a lui R @n raport cu ope-
ra\ia dat[.
5p c) Calcula\i (2016 )  (2015 )  (2014 )  . . .  2016  2017.

Editura Cardinal 59

 x 2y 

2. Fie M   3y x, y  Q, x  3y  1 .
2 2

 2
x 

8p a) Ar[ta\i c[ M este grup abelian @n raport cu @nmul\irea matricelor.
8p b) Ar[ta\i c[ G  x  y 3  x, y  Q, x 2  3y 2  1 este grup abelian @n
raport cu @nmul\irea numerelor.
x 2y
9p c) Ar[ta\i c[ f : G  M, f (x  y 3 )  3y este izomorfism @ntre cele
y
2
dou[ grupuri.

SUBIECTUL II (45 puncte)

1. Calcula\i:
5p a) 2x 2 1 dx;
cos x
5p b)  ln x  x 2  1 dx;

2e x  sin x  3 cos x .
2

10p c)  e x  sin x  2 cos x


0

1 1
2. Fie ]irul (I n ) n0 definit prin I 0  1 dx, I  x n dx pentru oricare n 
0 x 2
2 n
0 x 2
2

 N .
10p a) Calcula\i I 0  I 1 .
5p b) Studia\i monotonia ]irului (I n ) n0 .
10p c) Determina\i lim
n
(nI n ).

Lucrarea 2 - (timp de lucru: 2 ore) - Se acord[ 10 puncte din oficiu.

Alc[tuit[ de prof. RALUCA CIURCEA, Craiova

SUBIECTUL I (45 puncte)

10p 1. Realiza\i tabla @nmul\irii pe Z 8 ]i enumera\i elementele simetrizabile din Z 8


@n raport cu @nmul\irea.
log 7 (y1 )
5p 2. Demonstra\i c[ opera\ia x  y  1  (x  1 ) , definit[ pe (1,  ), este aso-
ciativ[.

60 Revista de matematic[ din Craiova


10p 3. Fie opera\ia x  y  xy  5x  5y  30, x, y  R ]i m  5, . Ar[ta\i c[ (M,  )
este monoid comutativ.
x 2  x 2  2016
10p 4. Calcula\i  dx.
x 2  2016
1
5p 5. Calcula\i  x 2  sin x dx.
1 x 2016  1
1
5p 6. Demonstra\i c[  1 dx  1.
0 x2  sin x  cos 2 x
2

SUBIECTUL II (45 puncte)

 1x 0 0 
 
1. Fie M   A(x )  x 1 x x  (1,  ) .
 0 0 1x 
 
5p a) Demonstra\i c[: A(x )  A(y )  A(x  y  xy ), ( ) x, y  (1,  ).
10p b) Demonstra\i c[ M este grup abelian @n raport cu @nmul\irea matricelor.
5p c) Calcula\i A(1 ) 2017 .
1 1
2. Fie ]irul (I n ) n0 definit prin I 0  1 dx ]i I  x n dx, ( )n  N  .
0 3x  4 n
0 3x  4

5p a) Calcula\i I 0 .
10p b) Demonstra\i c[ ]irul I n este m[rginit.
10p c) Afla\i lim I .
n n

*
* *

Editura Cardinal 61
************************************************************ *********
* PROBLEME PROPUSE *
*********************************************************************

GIMNAZIU

Clasa a V-a
G.1193. Determina\i mul\imile X ]i Y ]tiind c[ sunt @ndeplinite simultan condi\iile:
1) X  Y  0; 2012; 2013; 2014; 2015.
2) X  Y  2012; 2014; 2015.
3) Y  X N .
Prof. MIRCEA MARIO STOICA, Arad

G.1194. Compara\i numerele: 244 99 ]i 2186 69 .


Elev IONU|-FLORIN VOINEA, Bucure]ti

G.1195. S[ se determine n  N, astfel @nc`t 9 n  6 n  N.


n n

4 8
Prof. GRIGORE HORTOPAN, Rugi, Gorj
Prof. DORU-AURELIAN HORTOPAN, Rugi, Gorj

G.1196. Ar[ta\i c[ numerele ra\ionale 500n  600 , 600n  700 ]i 909n  712 nu
2009 2009 2009
pot fi simultan naturale, unde n este un num[r natural oarecare.
Prof. MIRCEA MARIO STOICA, Arad

G.1197. Determina\i numerele prime a, b, c ]tiind c[: 2016  7a  21b  299c.


Elev[ TEODORA M{RGINEANU, Bucure]ti

_______________________________________
Se primesc solu\ii p`n[ la 15.02.2017 (data po]tei)

62 Revista de matematic[ din Craiova


Clasa a VI-a
G.1198. Fie triunghiul isoscel ABC de baz[ BC. Perpendiculara @n A pe AB inter-
secteaz[ perpendiculara @n C pe BC @n D, iar perpendiculara @n A pe AC, intersecteaz[
perpendiculara @n B pe BC @n E. Dac[ DM  AC, M  AC, EN  AB, N  AB ]i
DM  EN  P, demonstra\i c[: a) triunghiul PBC este isoscel; b) AP  BC.
Prof. CONSTANTIN BASARAB, Craiova

G.1199. #n patrulaterul convex ABCD se ]tie c[: AD  BC ]i AB  CD.


Demonstra\i c[: AC  BD  BC  CD.
Prof. CARMEN TERHECI, Craiova
Prof. ROXANA VASILE, Craiova

G.1200. Fie a, b, c, d, e  Z, astfel @nc`t a 3  b 3  c 3  d 3  e 3  2016.


a) Ar[ta\i c[ cel pu\in unul din numere se divide cu 7.
b) Exist[ numere @ntregi x, y, z, t, u  Z, astfel @nc`t x 6  y 6  z 6  t 6  u 6  2016?
Prof. LIVIU SMARANDACHE, Craiova
Prof. DAN LUCIAN GRIGORIE, Craiova
G.1201. Determina\i toate numerele naturale cu trei cifre abc care sunt divizori ai
num[rului 2016 ]i satisfac rela\ia: a, (bc )  b, (ca )  c, (ab )  abc  bca  cab  37.
37
Elev[ TEODORA M{RGINEANU, Bucure]ti
G.1202. Dac[ a 1  2016, a 2  2015a 1  2016, a 3  2015a 2  2016 2 , . . ., a 2016 
 2015a 2015  2016 2015 , calcula\i a 2016 ]i ar[ta\i c[ S  a 1  a 2  . . .  a 2015 se divide cu
2016 31  1.
Elev[ TEODORA M{RGINEANU, Bucure]ti

Clasa a VII-a
G.1203. S[ se determine x, y  Z, astfel @nc`t y 6  x(x  1 )(x  2 )(x  3 )  40.
Prof. CONSTANTINA PRUNARU, Craiova
Prof. DAN GRIGORIE, Craiova
G.1204. Fie a, b, c, d, e, f  R, astfel @nc`t: (a  b  c ) 2  3(ab  bc  ca  d 2  e 2  f 2 ).
Ar[ta\i c[ a  b  c ]i d  e  f.
Prof. ROXANA VASILE, Craiova
Prof. TATIANA CRISTEA, Craiova

Editura Cardinal 63
G.1205. S[ se afle cifra x, ]tiind c[: 0, 11x  101
900
.
Prof. DELIA MITRAN, Timi]oara
G.1206. #n triunghiul ABC cu m A  90 , fie D proiec\ia lui A pe BC ]i M mijlocul

lui BC. #n exteriorul triunghiului se construie]te p[tratul BCFE ]i pe laturile BE,


EF, FC se iau punctele K, J, respectiv I, astfel @nc`t CD  BK  EJ  FI. Ar[ta\i c[
punctele A, M, J sunt coliniare dac[ ]i numai dac[ AB  AC.
Elev MIRCEA RAUL BODROGEAN, Bucure]ti

G.1207. #n paralelogramul ABCD fie M mijlocul lui AD, AC  BD  O, MC 


BD  P ]i AC  MB  Q. Ar[ta\i c[ dac[ G este centrul de greutate al triunghiu-
lui ABO, atunci QPGC este paralelogram.
Elev }TEFAN BURLACU, Bucure]ti

Clasa a VIII-a
G.1208. Fie n  N, n  2 ]i a 1 , a 2 , . . ., a n numere reale strict pozitive.
a a a a a a a  a n1 a n a n  a n a 1
Ar[ta\i c[: a 12  a 11 a 22  a 23  a 22 a 33  . . .  an1
n  a n1 a n
 a1  ana1 
a a a a
 a 12  a 23  . . .  an1
n
 a n1  n  1.
Prof. LUCIAN TU|ESCU, Craiova
Prof. LIVIU SMARANDACHE, Craiova
G.1209. Fie a, b, c  R. Ar[ta\i c[ a 4  b 4  c 4  a 2 b 2  b 2 c 2  c 2 a 2  a(b 3  c 3 ) 
b(a 3  c 3 )  c 3 (a  b ). #n ce caz avem egalitate?
Prof. SIMONA RADU ]i NINETA OPRESCU, Craiova
G.1210. Fie x, y, z  0 cu xyz  1 ]i m  n. Ar[ta\i c[:
(1  x m )(1  y m )(1  z m )  (1  x n )(1  y n )(1  z n ).
Prof. CAMELIA DAN{ ]i ILEANA DIDU, Craiova

G.1211. Fie numerele reale a, b, c ]i d astfel @nc`t a  b  c  d  0 ]i (a  b ) 


2

ab  cd  2. Demonstra\i inegalit[\ile urm[toare:


(i ) abcd  1. C`nd are loc egalitatea? (ii ) abcd   1 .
3
Prof. LEONARD GIUGIUC, Dr. Tr. Severin
Prof. DANIEL SITARU, Dr. Tr. Severin
G.1212. Determina\i numerele x, y, z, t  (0,  ) cu xyzt  1, ]tiind c[ are loc rela\ia:
2y 8t
x   4z   15 .
x  2 (x  2 )(y  2 ) (x  2 )(y  2 )(z  2 ) (x  2 )(y  2 )(z  2 )(t  2 ) 16
Prof. MIHAELA BERINDEANU, Bucure]ti

64 Revista de matematic[ din Craiova


LICEU

Clasa a IX-a
L.1213. Dac[ a, b, c, d, e sunt numere reale, astfel @nc`t a  b  c  d  e  abcde,
ar[ta\i c[: a 4  b 4  c 4  d 4  e 4  abcde.
Prof. LUCIAN TU|ESCU, Craiova
Prof. NICOLAE OPREA, B[lce]ti
  
L.1214. Exist[ @n plan vectorii a , b , c , astfel @nc`t rela\iile:
        
7 a  2b  c ; 7 b  2c  a ; 7 c  2a  b ,
s[ fie @ndeplinite simultan? Dar @n spa\iu?
Prof. LUCIAN TU|ESCU, Craiova
Prof. DAN GRIGORIE, Craiova

L.1215. Determina\i numerele reale x, y, z care satisfac condi\iile x 2  y 2  z 2  3 ]i


x 2  y 2  z 2  9  4 xy xy  yz yz  zx zx .
y z x
Prof. DAN SECL{MAN, Craiova

L.1216. Fie a, b, c  R, astfel @nc`t a  1  b  c ]i a  b  c  3. Demonstra\i ine-


galitatea (a 2  3a  1 )(b 2  3b  1 )(c 2  3c  1 )  1, ]i preciza\i c`nd are loc
egalitatea.
Prof. AMINE IDRISSI, Maroc
Prof. LEONARD GIUGIUC, Dr. Tr. Severin
Prof. DANIEL SITARU, Dr. Tr. Severin

L.1217. Dac[ a, b, c  (0, ) ]i (a  b  c )  a 2  b 2  c 2  2016, ar[ta\i c[:


2

ab  504  bc  504  ca  504  3.


ab bc ca
Prof. MIHAELA BERINDEANU, Bucure]ti

Editura Cardinal 65
Clasa a X-a
L.1218. Ar[ta\i c[: lg 2 7  lg 2 29  2  lg 4.
Prof. ILEANA DIDU, Craiova
Prof. CAMELIA DAN{, Craiova
L.1219. Decide\i dac[ exist[ numerele complexe z 1 , z 2 , z 3 , astfel @nc`t rela\iile:
7 z 1  2z 2  z 3 , 7 z 2  2z 3  z 1 , 7 z 3  2z 1  z 2 ,
s[ fie satisf[cute simultan.
Prof. LUCIAN TU|ESCU, Craiova
L.1220. Fie a  0. S[ se demonstreze c[: a  a  6 a  a  2.
3

Prof. CONSTANTIN GHEORGHE, Gr[di]tea, V`lcea


L.1221. Fie numerele reale a, b, c  1. Demonstra\i inegalitatea:
log b a  2 3 a  2  log c b  2 3 b  2  log a c  2 3 c  2  1.
Prof. DANIEL SITARU, Dr. Tr. Severin
Prof. MARIAN CUCOANE}, M[r[]e]ti, Vrancea
L.1222. Dac[ 1  a 1  a 2  . . .  a n , n  N  , ar[ta\i c[:
a
n
1 n
n(n  1 ) ln 2 a n1

i1 e ai  1
 ln 
i1
ai  n 
2
2

ln a 1  ln a n
.
Prof. CRISTIAN MOAN|{, Craiova

Clasa a XI-a
L.1223. a) Se consider[ numerele reale a, b, c  R ]i n, p  N , fixate. Calcula\i:
a 3 1  2015x p  b 3 1  2016x p  c 3 1  2017x p
lim xn .
x0
b) Enun\a\i ]i solu\iona\i o generalizare a problemei de la punctul a).
Prof. VALENTINA MIHAIU ]i DAN SECL{MAN, Craiova
L.1224. Fie A, B  M 2 (C ) dou[ matrice.
a) Dac[ TrA  0, ar[ta\i c[: (A 2 B  BA 2 )  O2  (AB  BA )  O2 .
2 2

b) Ar[ta\i c[: det(2AB  BA)  det AB  Tr(AB)  Tr(A2 B2 ).


2

Prof. FLORIN ST{NESCU, G[e]ti, D`mbovi\a


L.1225. Fie A, B  M 2 (R ) astfel @nc`t exist[ p, q  N, p  3, q  3 cu Ap  Bq  O2 .
Ar[ta\i c[: a) matricele A  B ]i A  B sunt singulare; b) (AB)  (BA)  O2 .
2 2

Prof. DAN SECL{MAN , Craiova


L.1226. Fie numerele reale pozitive a, b ]i c care satisfac condi\ia a 2  b 2  c 2  3.

66 Revista de matematic[ din Craiova


Demonstra\i inegalitatea: 1  1  1  1.
4a 4b 4c
Prof. MARIAN CUCOANE}, M[r[]e]ti, Vrancea
Prof. LEONARD GIUGIUC, Dr. Tr. Severin
L.1227. Fie f : RR o func\ie de dou[ ori derivabil[ ]i care verific[ proprietatea:
(f  (x )) 2  f  (x ) f (x ), ( ) x  R.
Ar[ta\i c[ pentru orice x1 , x2 , . . ., xn  R, n  N, n  2, exist[ rela\ia:
2 f (x 1 )  2 f (x 2 )  . . .  nf (x n )
 f (x 1 )  (f (x 2 ))  . . .  (f (x n ))  n(n1)  2 
2 n
.
n(n  1 )
Prof. CRISTIAN MOAN|{, Craiova

Clasa a XII-a
L.1228. Calcula\i  x  21  x ln x  cos(ln(x  1 ))dx, x  0.
2

x (x  1 )
Prof. AUREL CHIRI|{, Slatina
Prof. CONSTANTINA PRUNARU, Craiova
1
L.1229. Calcula\i I  t ln(1  t )(1  t 4 )dt.
0 1  t2
Prof. CAMELIA DAN{, Craiova
Prof. OANA PREDA, Craiova
L.1230. Fie f : RR o func\ie ce admite primitive ]i care are proprietatea c[ exist[
a, b  R, a  0, astfel @nc`t: f (f (x )  ax  b)  a f (x)  b, () x  R. Ar[ta\i c[ func\iile
g, h : RR, g(x)  min(f (x), ax  b), h(x)  max(f (x), ax  b), admit primitive pe R.
Prof. DAN SECL{MAN, Craiova
L.1231. Fie n un num[r natural, n  4 ]i numerele reale a 1 , a 2 , . . ., a p cu p  n, care
2n(n  1 )(n  2 )
satisfac rela\ia: (n  2 )   a i a j   a i a j a k  .
1ijn 1ijkn 3
n
Demonstra\i inegalitatea: (n  1 )   a p  2   aiaj.
p1 1ijn
Prof. LEONARD GIUGIUC ]i DIANA TR{ILESCU, Dr. Tr. Severin

L.1232. Fie f : 0, 1 R, o func\ie continu[ ]i concav[ pe o, 1, iar m  R, m  2,
1 1
 1 2  f (x )dx.
un num[r fixat. Demonstra\i c[:  f (x )dx  m 2 1  f (1 )  mm
1 m 0
m
Prof. CRISTIAN MOAN|{, Craiova
*
* *

Editura Cardinal 67
RUBRICA REZOLVITORILOR

CRAIOVA (jud. Dolj): C.N. “Carol I”, clasa a VII-a, prof. NICOLAE T{L{U: Gunescu
Corneliu (TG 4495-4519). Lic. Teoretic “Tudor Arghezi”, clasa a V-a, prof. DORINA
DR{CEA: }erban Alex (TG 4445-4469), Tra]c[ Mih[i\[ (TG 4445-4469), Badea Andrei
(TG 4445-4469), Preda Mihai (TG 4445-4469), clasa a VI-a, prof. DORINA DR{CEA:
Fota-B[rbulescu Alexandra (TG 4470-4494), Dragomir Anda (TG 4470-4494),
Cojoc[reanu Ania (TG 4471-4476, 4478, 4479), clasa a VII-a, prof. DORINA DR{CEA:
Buia Viviana (TG 4495-4519), Mitran Rebeca (TG 4495-4519), Ionescu Andrei (TG
4495-4519), Cre\u Maria (TG 4495-4519), Br[tan Alex (TG 4495-4519), P@rvulescu
Roxana (TG 4495-4519), Gruia Robert (TG 4495-4519), Ivanui Alexandru (TG
4495-4519), Gr[un\eanu Cristina (TG 4495-4519), Sandu Alexia (TG 4495-4519),
Drondoe Cornel (TG 4495-4519), Belciug Ianis (TG 4495-4519), Gruescu Riana (TG
4495-4519), Li\u R[zvan (TG 4495-4519), Modi Mihai (G 1163-1165). }c. Gim. “Traian”,
clasa a VI-a, prof. CONSTANTIN BASARAB: Ciurescu R[zvan Mihail (G 1154-1159,
1161, 1162). C.N. “Fra\ii Buze]ti”, clasa a V-a, prof. MARIA IONESCU: Cr[ciunescu
Matei (G 1153-1157), clasa a VI-a, prof. CLAUDIU CIULCU: Burheci Ioana (G
1158-1162), clasa a VIII-a, prof. CAMELIA DAN{: Ho]bot[ M[d[lina (TG 4520-4544),
clasa a IX-a, prof. LUCIAN TU|ESCU: Cr[ciunescu Emanuel (TL 3850-3874), clasa a
IX-a, prof. MIHAI DICU: Di\[ Alin Gabriel (TL 3850-3874), Pera Alexandru Lucian (TL
3850-3874), clasa a IX-a, prof. TEODORA R{DULESCU: Sandu George Cristian (TL
3850-3874), clasa a X-a, prof. MARIN POPA: Dr[ghia Denisa (TL 3875-3899), clasa a XI-a,
prof. OANA PREDA: Zanfir Bogdan (TL 3900-3924), Buzatu Alin (TL 3900-3924), clasa a
XII-a, prof. LUCIAN TU|ESCU: Nicolescu Andrei (L 1188-1192), P[tra]cu Cristina (L
1188-1192), Guri\[ Vladimir (L 1188-1192). }c. Gim. “Mircea Eliade”, clasa a III-a, prof.
HERMINA DINC{: Colan Mara Daria (III 1-25), clasa a VII-a, prof. MONICA MATEI:
Gavril[ Georgiana Elena (G 1163-1167). }c. Gim. “Mihai Viteazul”, clasa a VI-a, prof.
ZIZI BUTARU: Mitric[ Alessia-Georgiana (G 1158-1162). C.N. “}tefan Velovan”, clasa a
VII-a, prof. C{T{LIN CRISTEA: Smarandache Valeria Diana (TG 4470-4494). Palatul
Copiilor, clasa a VII-a, prof. VALERIA IOVAN: Diaconu-Dinu Antonia (G 1163-1167),
Ciurescu R[zvan (G 1163-1167). }c. cu clasele I-VIII, Nr. 29 “Nicolae Romanescu”, clasa a
..., @nv. LILIANA CUN{: Mihai Ionu\ (1-25), Ro]u R[zvan (1-25), Vancea David (1-25),
Vancea Albert (1-25), Teli\[ M[d[lina (1-25), clasa a ..., @nv. M{LINA R~}NI|{:
M[ce]anu Teodora (1-25), Oane R[zvan (1-25), Mitran Mario (1-25), Ciob[nescu
Bogdan (1-25), Ro]oga Elena (1-25), clasa a V-a, prof. CONSTANTIN CAZACU: Gu\[
Nolan (G 1153-1157), Popa Denisa (G 1153-1157), Veleanu Iasmina (G 1153-1157), clasa a
V-a, prof. SIDONIA CAZACU: Grecu Andreea (G 1153-1157), P[un Narcisa (G
1153-1157), Dinescu Eduard (G 1153-1157), clasa a VI-a, prof. CONSTANTIN CAZACU:
Nicola Sebastian (G 1158-1162), Nec]ulea }tefan (G 1158-1162), Nec]ulea Alexandru (G
1158-1162), clasa a VI-a, prof. MELANIA STOICULESCU: Petre Ana (G 1158-1162),
Cioabl[ Diana (G 1158-1162), clasa a VIII-a, prof. CONSTANTIN CAZACU: Tecu
Sebastian (G 1168-1172), Pisic[ Armanda Alexa (G 1168-1172), Nicoli\[ Andreea (G
1168-1172), clasa a VIII-a, prof. SIDONIA CAZACU: Ghera Theodora (G 1168-1172),

68 Revista de matematic[ din Craiova


Dobre Andrei (G 1168-1172), Dobre Robert (G 1168-1172), clasa a VIII-a, prof. MELANIA
STOICULESCU: Nu\[ Ana (G 1168-1172).
CUGIR (jud. Alba): }c. Gim. Nr. 3, clasa I, @nv. DORINA BR~NDU}E, DORIN
BR~NDU}E: Herlea Ianis (I 1-25), Pavel Andrei C[lin (I 1-25), Nicoar[ Daria (I 1-25),
Boruah Mara (I 1-25), Vulcu R[zvan (I 1-25), Rus Karina (I 1-25), P[troescu M[d[lina (I
1-25), Mih[ilesc Nicu]or (I 1-25), Breaz Alexandra (I 1-25), clasa a II-a , prof. @nv. primar
SABINA PRESECAN, @nv. ELISABETA HADA: Ivinis Andrei (II 1-25), Barbu Daniel
Eduard (II 1-25), Ple]a Adrian Nicolae (II 1-25), clasa a II-a, prof. ANDA TEC}A: Costea
Cristian (II 1-25), Grecu Iulian George (II 1-25), Laz[r Bogdan (II 1-25), Mara Bianca (II
1-25), Micu Sebastian (II 1-25), Molode\ Andrei Daniel (II 1-25), Zuga Daria Maria (II
1-25).
LUPENI (jud. Hunedoara): Lic. Teoretic “Mircea Eliade”, clasa a V-a, prof. EMILIA
VELCEA: Ilovan Adela (TG 4446-4450, 4453, 4455, 4459), clasa a VI-a, prof. EMILIA
VELCEA: Firescu Melisa (TG 4474, 4475, 4477, 4480, 4481), }oac[ Bianca (TG 4474,
4475, 4477, 4480, 4481, 4485), Beczuk Roberta (TG 4474, 4475, 4480, 4481, 4485), clasa a
VII-a, prof. EMILIA VELCEA: Neam\u Laura (TG 4495, 4499, 4504), Anghelu] Anca (TG
4495, 4499, 4504, 4506), clasa a VIII-a, prof. EMILIA VELCEA: Barna Andrei (TG 4495,
4504, 4527), Gherc[ Mirela (TG 4495, 4501, 4504, 4520, 4527), Sterie Darius (TG 4495,
4501, 4520, 4526, 4527). }c. Gen Nr. 2, clasa a IV-a, prof. ELENA DULEAN|U: Pal Adonis
(IV 1-25), Temelie Anca (IV 1-25), Duduianu M. Cristina (IV 1-12, 16-18, 24, 25), Bostan
Denisa (IV 1-4, 6, 16-18, 24, 25), Ciob[nescu Antonia (IV 1-20, 25).
TG. JIU (jud. Gorj): C. N. “Spiru Haret”, clasa a II-a, @nv. ILIE FLEANCU: Popescu
Maria (II 1-25), clasa a IX-a, prof. ION PIN|A: Popescu Ariana (TL 3850, 3854, 3864,
3874).
URECHE}TI (jud. Gorj): }c. Gim. Dr[gu\e]ti, clasa a II-a, prof. @nv. primar
VALENTINA-IRINA COVERC{: Balt[ Alexandru Valentin (II 1-25), B[r[gan Vasile
Alin (II 1-25), Eremia Irinel (II 1-25), Gu\[ Constantin-Georgian (II 1-25), Oproiu
Emilia-Teodora (II 1-25), Sanda Eugenia-Daniela (II 1-25), clasa a III-a, prof. @nv. primar
VALENTINA-IRINA COVERC{: Bercan Alexandra-Raimonda (III 1-25), Bumbu
Constantin-Ciprian (III 1-25), C[ld[ru]e Ana-Maria (III 1-25), Macovei Gabriel-Leonard
(III 1-25), clasa a IV-a, prof. @nv. primar COVERC{ VALENTINA-IRINA Gu\[ Maria (IV
1-25), Perne] Alexandra-Florentina (IV 1-25), Turturea Iustin-}tef[nu\-Octavian (IV
1-25).
SATU-MARE: C. N. “Doamna Stanca”, clasa a VI-a, prof. GIGEL BUTH: Balt[ Alex
Antonio (L 1178, 1180), clasa a IX-a, prof. GIGEL BUTH: Kadar Andreea (L 1173, 1175,
1178, 1180; G 1169, 1172), Lup]e Vanessa (L 1172, 1173, 1174; G 1167, 1168, 1169),
David Darius Nicolae (L 1173, 1174; G 1168, 1169), Megyesan Alexandra (L 1173, 1174;
G 1167-1169, 1172), Andreica Adrian Marius (L 1173, 1174; 1169), clasa a X-a, prof.
GIGEL BUTH: Medve David (L 1173; G 1169, 1167), clasa a XI-a, prof. GIGEL BUTH: Sas
Mariana (TL 3900-3913), Sas Daniela (TL 3900-3913), Belbe Ana (TL 3911, 3912).
ZIMNICEA (jud. Teleorman): Lic. Teoretic Zimnicea, clasa a IX-a, prof. TRAIAN
IANCULESCU: V[tafu Tiffany Monica (TL 3850-3856, 3860, 3861, 3863, 3865,
3866-3868, 3870-3872, 3874), Pu]ca] Denisa (TL 3850, 3854, 3855, 3860, 3866, 3868,
3871), |ancu Mihaela (TL 3850-3856, 3860, 3862, 3865, 3866, 3868, 3870, 3871, 3874),
clasa a XI-a, prof. TRAIAN IANCULESCU: Ianculescu Liviu Adrian (TL 3900, 3902, 3906,
3908, 3910-3914, 3923).

NOTA REDAC|IEI: TALON DE PARTICIPARE
Solu\iile problemelor
propuse se vor trimite pe Nume..........................................Prenume..................................
adresa redac\iei @nso\ite
de talonul de participare Localitatea.........................................Jud...................................
completat p`n[ la data de }coala.........................................................Clasa........................
15.02.2017.
Profesor.......................................................................................


I.S.S.N. 1221- 3551

Lei 10

S-ar putea să vă placă și